SFoM Summative 2

Ace your homework & exams now with Quizwiz!

E Most cri du chat (del 5p15) are not inherited.

A child, Jack, is born and within the first week, has a with high-pitched, shrill cry. Jack also has a low birth weight, diminished muscle tone (hypotonia), and microcephaly. Which is the most likely genetic transmission of this disease or syndrome? A. Autosomal recessive B. Autosomal dominant C. X-linked Recessive D. X-linked Dominant E. New Mutation

E

A couple have had difficulty having a child. They say that while they are able to conceive, the pregnancy fails early after conception. You order a karyotype for both the mother and father. Which parental karyotype best explains the situation? A. 45, XX, -12 +der[12q;13q] B. 46, XX, -12 +der[12q;13q] C. 45, XY, -13 +der[13q;14q] D. 46, XY, -13 +der[13q;14q] E. 45, XX, -13, -14, +der[13q;14q]

A

Apoptosis involves: A. a caspase cascade. B. hydrolysis of specific proteoglycans. C. microRNA cleavage. D. src kinase phosphorylation of a cdk (cyclin dependent kinase). E. release of cytochrome c from the Golgi.

E. The amniotic cavity develops within the epiblast, and it is a cavity that contains the embryo and amniotic fluid.

At what location does the amniotic cavity develop? (A) Between the cytotrophoblast and syncytiotrophoblast (B) Within the extraembryonic mesoderm (C) Between the endoderm and mesoderm (D) Within the hypoblast (E) Within the epiblast

B

C. Ardiac is a 32-yr-old man weighing 140 kg who has received a cardiac drug orally every day for the past month (100 mg/day, in tablet form). Recently, symptoms of toxicity appeared. Blood tests indicate that plasma concentrations of the drug are 100 ng/L (i.e. toxic), whereas they should be 12.5 ng/L. You stop drug treatment for two days, and re-measure drug levels; after this two-day hiatus, drug levels are 25ng/L. What new dosing regimen should you prescribe? A. Administer 12.5 mg/day immediately B. Withhold treatment for one additional day and then continue with 12.5 mg/day C. Withhold treatment for one additional day and then continue with 25 mg/day D. Withhold treatment for two additional days and then continue with 12.5 mg/day E. Withhold treatment for two additional days and then continue with 25 mg/day

C

Carol's father, Jack, was stricken with hemophilia. She had a son that also had hemophilia. Jack's only sibling is Jane. What is the likelihood that Carol's first cousin (female) on her father's side is a carrier for hemophilia? A. 1/16 B. 1/8 C. ¼ D. ½ E. 2/3

B

Consider the mosaic equation for membrane potential: Vm =(gK/gtot)•EK + (gCl/gtot)•ECl+ (gNa/gtot)•ENa If chloride is distributed at equilibrium across the membrane (ECl =Vm) and gCl > 0, which ONE of the following is true: A. Vm depends on the value of the chloride conductance, gCl. B. Vm does not depend on the value of the chloride conductance, gCl. C. The chloride current (iCl) is greater than zero D. The chloride current (iCl) is less than zero E. Vm = EK + ENa.

E One hormone/receptor complex can activate the gene transcription, resulting in multiple copies of the target mRNA. Similarly, one copy of mRNA can result in multiple copies of target protein, where the nuclear receptor hormone signaling is further amplified. All the other choices don't amplify the signal. They only relay the signal without signal amplification.

Cushing syndrome can be caused by excessive cortisol production by adrenal adenomas. When the cortisol molecules act on the target cells in the liver, muscle and fat tissue, what event(s) in the following amplifies the hormone signal? Find ONE correct answer. A. Delivery of cortisol to the target cells by serum binding proteins. B. Binding of cortisol to the glucocorticoid receptors. C. Dimerization of the glucocorticoid receptors. D. Binding of the dimeric receptors to the HREs. E. Production of messenger RNAs of the target genes.

D. During the later stages of pregnancy, the placental membrane becomes very thin and consists of two layers—the syncytiotrophoblast and the fetal endothelium.

During the later stages of pregnancy, maternal blood is separated from fetal blood by the (A) syncytiotrophoblast only (B) cytotrophoblast only (C) syncytiotrophoblast and cytotrophoblast (D) syncytiotrophoblast and fetal endothelium (E) cytotrophoblast and fetal endothelium

A. During week 2 of development, the embryoblast receives its nutrients from endometrial blood vessels, endometrial glands, and decidual cells via diffusion. Diffusion of nutrients does not pose a problem, given the small size of the blastocyst during week 2. Although the beginnings of a uteroplacental circulation are established during week 2, no blood vessels have yet formed in the extraembryonic mesoderm to carry nutrients directly to the embryoblast (this occurs in week 3).

During week 2 of development, the embryoblast receives its nutrients via (A) diffusion (B) osmosis (C) reverse osmosis (D) fetal capillaries (E) yolk sac nourishment

A

Females with trisomy X are: A. nearly normal and most are never ascertained. B. usually ascertained because of infertility issues. C. usually ascertained by phenotype at birth. D. usually ascertained by phenotype at puberty. E. usually spontaneously aborted.

B

Following formation of the zygote, which of the following statements is TRUE regarding cleavage, formation of the blastula, and implantation a. Cleavage normally does not occur while the zygote is transported along the uterine tube to reach the lumen of the body of the uterus b. Cleavage involves mitosis c. Implantation normally occurs in the uterine tube d. The zona pellucida ruptures after implantation has occurred

E

For the treatment of children with Disorders of Sexual Development, the Intersex Society of North America recommends: A. Children with intersex, parents of those children, and adults with intersex should be treated in an open, shame-free, supportive, and honest way. B. Children and adults with intersex, and their family members, should be provided with access to trained psychologists and social workers, especially when they are in distress (as some parents of newborns with intersex are). C. Following diagnostic work-up, newborns with intersex should be given a gender assignment as boy or girl, depending on which of those genders the child is more likely to feel as she or he grows up. D. Surgeries done to make the genitals look "more normal" should not be performed until a child is mature enough to make an informed decision for herself or himself. E. All of the above.

D All other choices are observed following stimulation of the parasympathetic system. Sympathetic system is associated with wide-spread and generalized response due to high pre-postganglion ratio (>1:10) and preganglionic neurons arise from the thoracolumbar region of the spinal cord.

Generally, responses produced by the sympathetic nervous system: A. conserve and restore energy. B. are localized and discrete. C. are mediated by cranial nerves. D. are wide-spread and generalized. E. are mediated by neurons in the sacral portion of the spinal cord.

D

Heartman is being treated with digoxin at a maintenance dose of 0.5 mg daily. He weighs 140 kg and presents early signs of digitalis toxicity. You reduce the dosage (0.4 mg/daily) and decide to keep the patient under observation. How long should you wait after reducing the dose before you take a blood sample for determining the new steady state level of serum digoxin? The half-life of digoxin in this patient is 1.5 hours. A. 1 hour B. 2 hours C. 4 hours D. 6 hours E. 24 hours

D. Tunica media = tunica muscularis

In this blood vessel, what layer is marked by the black asterisk? A. Tunica adventitia B. Tunica intima C. Tunica elastica D. Tunica muscularis E. Tunica arteriola

B

Li Fraumeni disease is: A. caused by a deficiency of a G-protein. B. a genetic disorder involving a tumor suppressor gene. C. characterized by overproduction of the Lifrau protein. D. caused by down regulation of an anti-oncogene, receptor tyrosine kinase. E. initiated by chemical carcinogenesis.

A

Mary and her half-sister (different mother, same father) have both had boys affected with Duchenne's muscular dystrophy (DMD). The father is not affected. This is most likely an example of: A. Germ-line mosaicism B. New mutations during meiosis C. Incomplete penetrance D. Skewed X-inactivation E. X-autosomal translocation

C - capillaries are associated in gas exchange

Most of the exchange of oxygen and carbon dioxide in the tissues of the body occurs in association with the vessels (at the arrows) pictured in which of the labeled panels? A. A B. B C. C D. D E. E

C. Myotonic: anticipation 30-80 father; 80+ mother Fragile-X: anticipation w/ mother Huntington: anticipation w/ father

Myotonic dystrophy, Fragile-X, and Huntington Disease are all diseases: A. caused by poly Gln inserts within their protein products. B. that are typically adult onset disorders. C. that all contain mutant expansions that can be further amplified D. caused by mutations that decrease the expression of the mutant allele. E. associated with chromosomal fragile sites at the expansion sites

B

Perhaps the most critical gene in the early development of colorectal cancer (CRC) is the APC tumor suppressor gene. Somatic mutations in both alleles are present in 80% of sporadic CRCs, and a single germline mutation in this gene is responsible for familial adenomatous polyposis (FAP), a dominantly inherited syndrome characterized by the development of hundreds to thousands of colorectal polyps by the second or third decade of life. These APC mutations result in: A. inactivation of the Wnt signaling pathway leading to gut stem cell apoptosis. B. activation of the Wnt signaling pathway leading to proliferation of gut stem cells. C. activation of the NF-κB signaling pathway, which inhibits apoptosis and promotes cell proliferation. D. activation of the ras signaling pathway to promote production of cyclin D. E. inactivation of the ras signaling pathway to promote apoptosis.

D In contrast to the Pertussis toxin, cholera toxin works by inhibiting the hydrolysis of GTP by Gαs.

Pertussis toxin interferes with the cytokine release by A. inhibiting hydrolysis of GTP by Gαi. B. inhibiting hydrolysis of GTP by Gαs. C. inhibiting GDP/GTP exchange on Gαs. D. inhibiting GDP/GTP exchange on Gαi. E. inhibiting GDP/GTP exchange on Gαq.

D Phospholipase C (PLC) cleaves PIP2 → IP3 & DAG (PLC hydrolyzes the glycerophosphate bond) IP3 triggers calcium release from ER

Phospholipase-catalyzed hydrolysis of which of the following compounds results in an increased cytosolic concentration of calcium ions in response to ligand binding to members of the serotonin family? A. phosphatidylcholine. B. phosphatidylethanolamine C. phosphatidylglycerol D. phosphatidylinositol bisphosphate E. phosphatidylserine

D Vd for him=2L Vd=loading dose/[drug] in plasma 2=loading/10 20mg, but after passage of 2 half lives --> 80 mg

Reed is 5-year-old child who tends to eat everything he finds around. He arrives, yet again, to the emergency room, showing signs of severe respiratory depression, after having ingested a large amount of fast-acting opiate-containing anti-pain medication 2 h before. You immediately measure opiates levels, and find that plasma concentrations are 10 mg/L. If Reed weighs 35 kg, how many mg of opiates did he ingest 2h before? Note that the opiates he ingested follow first-order kinetics with t½ of 1h (in all individuals, including Reed) and with Vd=4L in adult individuals. A. 10 mg B. 20 mg C. 40 mg D. 80 mg E. 100 mg

A

Region 2 highlights the muscularis, which is A. smooth muscle only. B. skeletal muscle only. C. smooth and skeletal muscle mixed together.

C bc only smooth muscle is shown.

Region 2 highlights the muscularis, which is smooth muscle only. Based on these observations, the region of the esophagus shown is A. upper esophagus. B. middle exophagus. C. lower esophagus.

B

Suppose that a cell utilizes a Na+ /H+ counter-transporter to extrude H+. Would this countertransporter be able to extrude more efficiently H+ if the membrane potential was clamped at -85 mV rather than its usual -60 mV? A. Yes B. No

C

The figure below represents the plasma concentrations of pharmagood, a new drug developed to promote critical thinking. Pharmagood is given as a bolus intravenous injection of 2048mg in a normal individual (a pharmacology student, weighing 70kg). The empty circles represent plasma concentrations of pharmagood in its alpha phase, whereas the filled circles represent concentrations of pharmagood during its beta phase (before or after the administration of sodium bicarbonate). What is the approximate clearance rate AFTER the administration of sodium bicarbonate? A. 0.175 L/h B. 0.35 L/h C. 0.7 L/h D. 1.4 L/h E. 2.8 L/h

D Each generation loses 1/3 of the mutant alleles. If the incident rates remain the same over time, then 1/3 of patients must have a sporadic mutation.

The incidence of an X-linked disease with zero fitness remains constant over time. Which of the following best represents the percentage of patients, whose defective disease causing allele arose from sporadic rather than inherited mutation? A. 10% B. 20% C. 25% D. 33% E. 50%

A. The intervillous space contains only maternal blood as the spiral arteries of the endometrium penetrate the outer cytotrophoblast shell.

The intervillous space of the placenta contains (A) maternal blood (B) fetal blood (C) maternal and fetal blood (D) amniotic fluid (E) maternal blood and amniotic fluid

D Huntington's with reduced penetrance Achondroplasia & Marfran's are 100% penetrant

This above pedigree is most consistent with the disease: A. Duchene's muscular dystrophy. B. Achondroplasia C. Marfan's D. Huntington's E. Fragile X

C

This disease is caused by is caused by a loss of function mutation but is inherited in a dominant manner. A. Tay Sachs B. Huntington's disease C. Familial hypercholesterolemia D. Neurofibromatosis E. β-thalassemia F. Rett syndrome G. Muscular dystrophy H. Cystic Fibrosis I. 45, X J. 47, XXY K. 47, XXX L. 47, XYY M. 47, XY, +13 N. 47, XX, +18 O. 47, XY, +21

I

Turner Syndrome A. Tay Sachs B. Huntington's disease C. Familial hypercholesterolemia D. Neurofibromatosis E. β-thalassemia F. Rett syndrome G. Muscular dystrophy H. Cystic Fibrosis I. 45, X J. 47, XXY K. 47, XXX L. 47, XYY M. 47, XY, +13 N. 47, XX, +18 O. 47, XY, +21

D

US citizens whose ancestors originated from equatorial countries, including Africa, have a higher incidence of sickle cell disease and thalassemias illustrating: A. autosomal dominant inheritance. B. negative selection. C. the founder effect. D. the heterozygote advantage. E. X-linked inheritance

D

What is the likelihood that the proband is a carrier for the disease mutation? A. 100% B. 66% C. 50% D. 25% E. 12.5%

1/8 ½ x ½ x ½ = 1/8

What is the likely-hood Sandra's first child is affected by this disease? (from above)

Nerves

What is the structure marked by the orange asterisk?________________

B. Primordial germ cells, the predecessors to gametes, are first seen in the wall of the yolk sac at week 4 of embryonic development, and they migrate into the gonads at week 6.

Where do primordial germ cells initially develop? (A) In the gonads at week 4 of embryonic development (B) In the yolk sac at week 4 of embryonic development (C) In the gonads at month 5 of embryonic development (D) In the yolk sac at month 5 of embryonic development (E) In the gonads at puberty

D Nuclear receptors have a DNA binding domain between the transcription domain and the hormone binding domain.

Which one of the following will bind to the hormone response element (HRE) of DNA? A. Adrenocorticotropic hormone (ACTH). B. Cortisol. C. protein kinase A. D. Androgen receptor. E. G-protein coupled receptor.

A

Which one of the karyotypes below is associated with a phenotype that is most likely to be preliminarily diagnosed at live birth? A. 45, X B. 47, XXX C. 47, XYY D. 47, XXY E. 46, XY female

A

Which one of the phenotypes pictured below is caused by an autosomal dominant, 100% penetrant, gain-of-function mutation with a high number of new mutations?

B

p53 is called the "guardian of the genome" and: A. is a tetramer of 4 subunits of 13,250 Da each, for a total of 53,000 Da. B. is activated by phosphorylation. C. regulates the cell cycle at the G2/M interface through p27 cdk inhibitor. D. is a translational factor that regulates protein synthesis. E. has mutational 'hot spots' that activate it in tumors.

E

A special histological stain was used on the specimen shown in the image below. The small yellow asterisk is surrounded by black wavy structures. They are A. type I collagen fibers. B. reticular fibers. C. artifacts of tissue preparation. D. myofibrils. E. elastic fibers.

D

A special histological stain was used on the specimen shown in the image below. The structure containing the large yellow asterisk in its lumen is a(n) A. medium-sized vein B. arteriole C. venule D. muscular artery E.. esophagus

C

A 1 week-old newborn has microcephaly, features presenting as a broad bridge of the nose continuing to the forehead with a larger than normally spacing between the eyebrows and above the nose, seizures, and apparent hearing loss. Which of the following kaytotypes is most consistent with these disease phenotypes? A. 45, X B. 46, XX C. 46, XX, del (4p16) D. 46, XX, t(14;21) E. 46, XY F. 47, XX, +13 G. 47, XX, +18 H. 47, XX, +21 I. 47, XX, +22

A

A 1 year old child, with ambiguous genitalia, that did not receive the Illinois mandated newborn screening is admitted into the emergency room with salt wasting, failure to thrive, recurrent vomiting, dehydration, hypotension, hyponatremia, hyperkalemia, hypoglycemia, and shock (classic salt-wasting adrenal hyperplasia). A deficiency in which of the following is most consistent with this illness? A. 21-hydroxylase (CYP21) B. 17, 20 Lyase (CYP17A1) C. 17-α-hydroxylase (CYP17A1) D. 18-hydroxylase (CYP11B2) E. the androgen receptor

C

A 11 year old girl born of second degree consanguineous marriage presented with history of swaying while walking for past two years. There was associated difficulty in squatting and climbing downstairs. There was no history of tripping while walking, wasting of muscles tingling or paraesthesia in the lower limbs. Over one year it slowly progressed to gait instability. She did not have any feeding difficulties. There was no significant past or family history. Examination revealed high arched palate and pes cavus (high arches in feet). What is the best diagnosis for this patient? A. Spinal bulbar muscular dystrophy. B. Myotonic dystrophy C. Friedreich ataxia D. Fragile X E. Duchenne muscular dystrophy

C DiGeorge syndrome is an immunodeficiency characterized by infection, a small thymus, congenital malformations, and hypoparathyroidism. This cluster is characteristic of the 22q11.2 deletion syndrome, readily diagnosed by FISH. Ade- nosine deaminase deficiency can cause immunodeficiency, but it is not associated with congenital malformations. Branched DNA assay can detect HIV infection that can lead to AIDS, but no congenital anomalies are associated with this condition. A lymph node biopsy may show a reduction in T cells or B cells associated with various forms of immunodeficiency, but this is not a specific test that can aid in confirming a specific di- agnosis. Trinucleotide repeats of the X chromosome, detected by PCR, are seen in fragile X syndrome, which manifests with mental retardation in males.

A 12-year-old boy has a cough and earache for the past 2 days. He has a history of recurrent infections, including otitis media, diarrhea, and pneumonia. Physical examination shows an erythematous right tympanic membrane, a cleft palate, and murmur suggestive of congenital heart disease. A thoracic CT scan shows a small thymus. Results of laboratory studies suggest mild hypoparathyroidism. Which of the following diagnostic studies is most likely to be helpful in diagnosing this patient's condition? A Adenosine deaminase assay in lymphocytes B Branched DNA assay for HIV-1 RNA level C FISH analysis with a probe for chromosome 22q11.2 D Lymph node biopsy E PCR analysis for trinucleotide repeats on the X chromosome

D Type 1 diabetes mellitus has an increased frequency in some families, but the exact mechanism of inheritance is unknown. The risk is increased for offspring when first-order relatives are affected. HLA-linked genes and other genetic loci and environmental factors are considered important. This pat- tern of inheritance is multifactorial. The other listed inheritance patterns are not seen with most cases of diabetes mellitus.

A 13-year-old boy has been drinking large quantities of fluids and has an insatiable appetite. He is losing weight and has become more tired and listless for the past month. Labora- tory findings include normal CBC and fasting serum glucose of 175 mg/dL. His parents, two brothers, and one sister are healthy. A maternal uncle is also affected. Which of the follow- ing is the probable inheritance pattern of his disease? A Autosomal dominant B Autosomal recessive C Mitochondrial DNA D Multifactorial E X-linked recessive

A Neurofibromatosis type 1 (NF-1) is characterized by the development of multiple neurofibromas and pigmented skin lesions. Neurofibromas are most numerous in the der- mis but also may occur in visceral organs. Patients with NF-1 also may develop a type of sarcomatous neoplasm known as a malignant peripheral nerve sheath tumor (MPNST). NF-1 is a tumor suppressor that appears with an autosomal dominant pattern of inheritance, though some cases result from spontaneous new mutations (no prior family members with the mutation). NF-1 exhibits variable expressivity, because the manifestations (location and types of neoplasms) are not the same in all patients. The other forms of inheritance listed are not associated with tumor suppressor genes.

A 15-year-old girl has developed multiple nodules on her skin over the past 10 years. On physical examination, there are 20 scattered, 0.3-cm to 1-cm, firm nodules on the patient's trunk and extremities. There are 12 light brown macules aver- aging 2 to 5 cm in diameter on the skin of the trunk. Slit-lamp examination shows pigmented nodules in the iris. A sibling and a parent are similarly affected. Genetic analysis shows a loss-of-function mutation. Which of the following inheritance patterns is most likely to be present in this family? A Autosomal dominant B Autosomal recessive C Mitochondrial D Multifactorial E X-linked recessive

C The liver is one of the few organs in the human body that can partially regenerate. This is a form of compensatory hyperplasia. The stimuli to hepatocyte mitotic activity cease when the liver has attained its normal size. Hepatocytes can reenter the cell cycle and proliferate to regenerate the liver; they do not just hypertrophy (increase in size). Apoptosis is single cell death and frequently occurs with viral hepatitis. Dysplasia is disordered epithelial cell growth that can be premalignant. Hydropic change, or cell swelling, does not produce regeneration. Steatosis (fatty change) can lead to hepatomegaly, but not as a regenerative process. It is the result of toxic/metabolic hepatocyte injury.

A 16-year-old boy sustained blunt trauma to his abdo- men when he struck a bridge abutment at high speed while driving a motor vehicle. Peritoneal lavage shows a hemoperi- toneum, and at laparotomy, a small portion of the left lobe of the injured liver is removed. Two months later, a CT scan of the abdomen shows that the liver has nearly regained its size before the injury. Which of the following processes best ex- plains this CT scan finding? A Apoptosis B Dysplasia C Hyperplasia D Hydropic change E Steatosis

D. Breast lobules have an increased number of cells under hormonal influence (mainly progesterone) to provide for normal lactation. Ductal metaplasia in the breast is a pathologic process. Epithelial dysplasia denotes disordered growth and maturation of epithelial cells that may progress to cancer. Accumulation of fat within cells is a common manifestation of sublethal cell injury or, uncommonly, of inborn errors in fat metabolism. The breast stroma plays no role in lactation and may increase with pathologic processes.

A 20-year-old woman breastfeeds her infant. On examination, her breasts are slightly increased in size. Milk can be expressed from both nipples. Which of the following processes that occurred in her breasts during pregnancy enables her to breastfeed the infant? A. ductal metaplasia B. epithelial dysplasia C. intracellular lipid deposition D. lobular hyperplasia E. stromal hypertrophy

D In cystic fibrosis the elevated sweat chloride level is related to a defect in the transport of chloride ions across epi- thelia. The most common genetic defect is a deletion of three base pairs at the ∆F508 position coding for phenylalanine in the cystic fibrosis transmembrane conductance regulator (CFTR) gene. A frameshift mutation involves one or two base pairs, not three, and changes the remaining sequence of amino acids in a protein. A point mutation may change the codon to the sequence of a "stop" codon, which truncates the protein being synthesized, typically leading to degradation of the protein. A point mutation typically is a missense mutation that leads to replacement of one amino acid for another in the protein chain; this can lead to abnormal conformation and function of the protein. A trinucleotide repeat sequence mutation leads to amplification of repeats of three nucleotides, so-called tandem repeats, which prevent normal gene expression.

A 21-year-old primigravida gives birth to a term infant after an uncomplicated pregnancy. The infant is of normal height and weight, and no anomalies are noted. The infant fails to pass meconium. Laboratory studies show an elevated sweat chloride level. Genetic testing indicates that a critical protein coded by a gene is missing one phenylalanine amino acid in the protein sequence. Which of the following types of gene mutations is most likely to produce these findings? A Frameshift B Nonsense (stop codon) C Point D Three-base pair deletion E Trinucleotide repeat

B. The posterior neuropore closes during week 4 (day 27). Failure of the posterior neuropore to close results in lower neural tube defects, such as spina bifida.

A 22-year-old pregnant woman at 20 weeks of gestation comes to her OB/GYN for a normal examination. During routine blood tests, her serum α-fetoprotein (AFP) concentration is markedly increased for her gestational age. Ultrasonography reveals spina bifida in the fetus. At what week of gestation did this defect most likely occur? (A) 1-2 (B) 4-6 (C) 9-11 (D) 12-15 (E) 16-19

D. The increase in uterine size is primarily the result of an increase in the size of myometrial smooth muscle cells. The endometrium also increases in size, mainly via hyperplasia, but it remains as a thing lining to the muscular wall and does not contribute as much to the change in size. There is little stroma in myometrium and a greater proportion in endometrim, so stroma contributes a smaller percentage to the gain in size than muscle. The vessels are a minor but essential component in this increase in size, but not the largest component.

A 22-year-old woman becomes pregnant. A fetal ultrasound examination at 13 weeks' gestation shows her uterus measures 7x4x3 cm. At deliver of a term infant, her uterus measures 34x18x12 cm. Which of the following cellular processes has contributed most to the increase in her uterine size? A. endometrial glandular hyperplasia B. myometrial fibroblast proliferation C. endometrial stromal hypertrophy D. myometrial smooth muscle hypertrophy E. vascular endothelial hyperplasia

A There is an apoptotic cell (arrow) that is shrunken and has been converted into a dense eosinophilic mass. There is a surrounding inflammatory reaction with cytotoxic lympho- cytes. This pattern is typical of apoptosis. Caspase activation is a universal feature of apoptosis, regardless of the initiat- ing cause. Apoptosis induced in recipient cells from donor lymphocytes occurs with graft-versus-host disease. Lipases are activated in enzymatic fat necrosis. Reduced ATP synthe- sis and increased glycolysis occur when a cell is subjected to anoxia, but these changes are reversible. Lipid peroxidation occurs when the cell is injured by free radicals.

A 22-year-old woman with leukemia undergoes bone marrow transplantation and receives a partially mismatched donor marrow. One month later, she has a scaling skin rash. A skin biopsy is obtained, and on microscopic examination, it has the cellular change shown in the figure. This change most likely results from which of the following biochemical reactions? A Activation of caspases B Elaboration of lipases C Increase in glycolysis D Peroxidation of lipids E Reduction of ATP synthesis

C Physicians must be cautious in assigning sex to an infant with ambiguous genitalia; changing one's opinion is about as popular as an umpire changing the call. True her- maphroditism, with ovarian and testicular tissue present, is very rare. This infant has female pseudohermaphroditism, resulting from exposure of the fetus to excessive androgenic stimulation, which in this case is due to congenital adrenal hyperplasia. The gonadal and the karyotypic sex are female. Male pseudohermaphroditism has various forms, but the most common is androgen insensitivity; affected individu- als are phenotypically females, but have testes and a 46,XY karyotype. Nondisjunctional events lead to monosomies or trisomies, and most result in fetal loss. Trinucleotide repeats are seen in males with fragile X syndrome. Abnor- malities of mitochondrial DNA have a maternal transmis- sion pattern and do not involve sex chromosomes or sexual characteristics.

A 23-year-old woman gives birth to a term infant after an uncomplicated pregnancy. On physical examination, the infant has ambiguous external genitalia. The parents want to know the infant's sex, but the physician is hesitant to assign a sex without further information. A chromosomal analysis indicates a karyotype of 46,XX. An abdominal CT scan shows bilaterally enlarged adrenal glands, and the internal genitalia appear to consist of uterus, fallopian tubes, and ovaries. This clinical pic- ture is most consistent with which of the following conditions? A Androgen insensitivity syndrome B Excessive trinucleotide repeats C Female pseudohermaphroditism D Mitochondrial DNA mutation E Nondisjunctional event with loss of Y chromosome

B The small, discrete nature of this mass and its slow growth with nearly unchanged size suggest a benign neoplasm. The red color is consistent with vascularity. A hemangioma is a common benign lesion of the skin. Fibroadenomas arise in the breast. Leiomyomas, which are white, arise from smooth muscle and are most common in the uterus. Lipomas are yellow fatty tumors that can occur beneath the epidermis. Melanomas are malignant and tend to increase in size quickly; many are darkly pigmented. The benign counterpart to the melanoma is the nevus, which is quite common, but nevi are usually light to dark brown.

A 23-year-old woman has noted a nodule on the skin of her upper chest. She reports that the nodule has been present for many years and has not changed in size. It is excised, and the microscopic appearance is shown in the figure. Which of the following neoplasms is this lesion most likely to be? A Fibroadenoma B Hemangioma C Leiomyoma D Lipoma E Melanoma F Nevus

The answer is B: Dystrophic calcifi cation. Dystrophic calcification reflects underlying cell injury. Serum levels of calcium are normal, and the calcium deposits are located in previously damaged tissue. Intrauterine Toxoplasma infection affects approximately 0.1% of all pregnancies. Acute encephalitis in the fetus afflicted with TORCH syndrome may be associated with foci of necrosis that become calcified. Microcephaly, hydrocephalus, and microgyria are frequent complications of these intrauterine infections. Metastatic calcifi cation (choice D) reflects an underlying disorder in calcium metabolism.

A 24-year-old woman contracts toxoplasmosis during her pregnancy and delivers a neonate at 37 weeks of gestation with a severe malformation of the central nervous system. MRI studies of the neonate reveal porencephaly and hydrocephalus. An X-ray film of the head shows irregular densities in the basal ganglia. These X-ray findings are best explained by which of the following mechanisms of disease? (A) Amniotic fluid embolism (B) Dystrophic calcification (C) Granulomatous inflammation (D) Metastatic calcification (E) Organ immaturity

A Technically, this answer is wrong because leiomyoma is a benign tumor of muscle tissue. A leiomyoma, also known as fibroids, is a benign smooth muscle tumor that very rarely becomes cancer (0.1%). They can occur in any organ, but the most common forms occur in the uterus, small bowel, and the esophagus. Lipoma would be mesenchymal in origin.

A 24-year-old woman with a history of heavy and painful menstrual periods has been having difficulty conceiving despite months of trying to become pregnant. Further workup includes a bimanual pelvic examination and an ultrasound, which demonstrates a mass in the uterus that is presumed to be a leiomyoma. This mass is a (A) benign tumor of mesenchymal tissue. (B) benign tumor of surface epithelium. (C) malignant tumor of epithelial tissue. (D) malignant tumor of glandular epithelium. (E) malignant tumor of mesenchymal tissue.

D. Reduced AFP levels are associated with Down syndrome. All of the other defects (neural tube defects such as spina bifida and anencephaly, omphalocele, and esophageal atresias) are associated with elevated AFP levels.

A 25-year-old pregnant woman at 17 weeks of gestation comes to her OB/GYN for a normal examination. During routine blood tests, her serum α-fetoprotein (AFP) concentration is found to be markedly decreased for her gestational age. Which abnormality will the physician need to rule out based on these low AFP levels? (A) Spina bifida (B) Anencephaly (C) Omphalocele (D) Down syndrome (E) Esophageal atresia

A The patient has a classic history of familial adenomatous polyposis with numerous adenomatous polyps and malignant transformation. The earliest event in the APC → adenocarcinoma sequence is loss of APC gene function. This prevents the destruction of β-catenin in the cytoplasm, which translocates to the nucleus and coactivates transcription of several genes. The APC → β-catenin sequence is a component of the WNT signaling pathway. RAS activation occurs after the malignant transformation sequence is initiated by the APC (gatekeeper) gene. Loss of cell cycle G1 arrest occurs with p53 loss late in the sequence. Mutations in mismatch repair genes give rise to hereditary nonpolyposis colon cancer syndrome from loss of ability to repair DNA damage. The BCL2 gene is not involved in the transition from adenoma to carcinoma.

A 26-year-old man with a family history of colon carcinoma undergoes a surveillance colonoscopy. It reveals hundreds of polyps in the colon, and two focal 0.5-cm ulcerated areas. A biopsy specimen from an ulcer reveals irregularly shaped glands that have penetrated into the muscular layer. Which of the following molecular events is believed to occur very early in the evolution of his colonic disease process? A Activation of the WNT signaling pathway B Inability to hydrolyze GTP-bound RAS C Loss of heterozygosity affecting the p53 gene D Mutations in mismatch repair genes. E Translocation of BCL2 from mitochondria to cytoplasm

B Approximately 5% to 10% of breast cancers are familial, and 80% of these cases result from mutations in the BRCA1 and BRCA2 genes. Onset of these familial cancers occurs earlier in life than the sporadic cancers. The protein products of these genes are involved in DNA repair. BCL2 is overexpressed in some lymphoid neoplasms. The epithelial growth factor receptor ERBB2 (HER2) overexpression is present in some sporadic breast cancers; other EGF alterations can be seen in lung, bladder, gastrointestinal, ovarian, and brain neoplasms. The HST1 mutation is seen in some gastric cancers. IL2 overexpression is associated with some T cell neoplasms. K-RAS overexpression is seen in many cancers, including some breast cancers, but the early age of onset and family history in this case strongly suggest BRCA mutations.

A 26-year-old woman has a lump in her left breast. On physical examination, she has an irregular, firm, 2-cm mass in the upper inner quadrant of the breast. No axillary adenopa- thy is noted. A fine-needle aspirate of the mass shows anaplas- tic ductal cells. The patient's 30-year-old sister was recently diagnosed with ovarian cancer, and 3 years ago her maternal aunt was diagnosed with ductal carcinoma of the breast and had a mastectomy. Mutation involving which of the following genes is most likely present in this family? A BCL2 (anti-apoptosis gene) B BRCA1 (DNA repair gene) D ERBB2 (growth factor receptor gene) E HST1 (fibroblast growth factor gene) F IL2 (growth factor gene) G K-RAS (GTP-binding protein gene)

D The liver has a dual blood supply, with a hepatic arterial circulation and a portal venous circulation. Infarction of the liver caused by occlusion of hepatic artery is uncommon. Cerebral infarction typically produces liquefactive necrosis. Infarcts of most solid parenchymal organs such as the kidney, heart, and spleen exhibit coagulative necrosis, and emboli from the left heart often go to these organs.

A 28-year-old woman with a 15-year history of recur- rent thrombosis from a prothrombin gene mutation develops septicemia after a urinary tract infection with Pseudomonas aeruginosa. She develops multiple infarcts and organ failure over the next 2 weeks. Which of the following organs is most likely to be spared from the effects of ischemic injury in this woman? A Brain B Heart C Kidney D Liver E Spleen

B. Cystic fibrosis.

A 29 year-old woman has a history of steatorrhea and recurrent pulmonary infections since childhood. She experiences a fracture, and the radiograph shows osteopenia. Laboratory studies show an abnormal sweat chloride level. Neither parents nor siblings are affected. Genetic studies show a mutation in a gene encoding for chloride ion channel. Which of the following inheritance patters is her diseases most likely to have? A. autosomal dominant B. autosomal recessive C. mitochondrial DNA D. multifactorial E. X-linked recessive

A Caseous necrosis is part of granulamatous inflammation

A 29-year-old man hospitalized for acquired immunodeficiency syndrome (AIDS) is found to have pulmonary tuberculosis. Which type of necrosis is found in the granulomatous lesions (clusters of modified macrophages) characteristic of this increasingly frequent complication of AIDS? (A) Caseous (B) Coagulative (C) Enzymatic (D) Fibrinoid (E) Liquefactive

B Reduced workload causes cell to shrink through loss of cell substance, a process called atrophy. The cells are still present, just smaller. Aplasia refers to lack of embryonic development; hypoplasia describes poor or subnormal development of tissues. Dystrophy of muscles refers to inherited disorders of skeletal muscles that lead to muscle fiber destruction, weakness, and wasting. Hyaline change (hyalinosis) refers to a nonspecific, pink, glassy eosinophilic appearance of cells.

A 29-year-old man sustains a left femoral fracture in a mo- torcycle accident. His leg is placed in a plaster cast. After his left leg has been immobilized for 6 weeks, the diameter of the left calf has decreased in size. This change in size is most likely to result from which of the following alterations in his calf muscles? A Aplasia B Atrophy C Dystrophy D Hyalinosis E Hypoplasia

B She has cystic fibrosis, due to inheritance of two abnormal alleles for the cystic fibrosis transmembrane conductance regulator (CFTR) gene. There are multiple mutations, which may have different degrees of severity. The incidence in the United States is about 1 in 2500 live births, from a carrier rate of 1 in 25 (this example makes the math easy). Autosomal dominant mu- tations tend to involve 50% of family members, and structural proteins tend to be affected. Mitochondrial DNA is inherited on the maternal side and affects mainly genes associated with oxidative phosphorylation. Multifactorial inheritance does not have a well-defined recurrence risk, but tends to run in fami- lies, and is more characteristic for diseases such as diabetes mellitus or schizophrenia. X-linked recessive disorders are most likely to appear in males born to female carriers.

A 29-year-old woman has a history of steatorrhea and recurrent pulmonary infections since childhood. She experi- ences a fracture, and the radiograph shows osteopenia. Labo- ratory studies show an abnormal sweat chloride level. Neither parents nor siblings are affected. Genetic studies show a mu- tation in a gene encoding for chloride ion channel. Which of the following inheritance patterns is her disease most likely to have? A Autosomal dominant B Autosomal recessive C Mitochondrial DNA D Multifactorial E X-linked recessive

B

A 3-year-old boy has excessive eating disorder, delayed milestones motor and language skills, and is shorter than average. Which of the following is the most likely diagnosis? A. Achondroplasia B. Prader Willi syndrome C. Angelman's syndrome D. Williams syndrome. E. Congenital adrenal hyperplasia.

A The most common tumor of infancy is a hemangioma, and these benign neoplasms form a large percentage of childhood tumors as well. Although benign, they can be large and disfiguring. Fibromatoses are fibromatous prolifer- ations of soft tissues that form solid masses. Lymphangioma is another common benign childhood tumor seen in the neck, mediastinum, and retroperitoneum. Lymphoblasts as part of leukemic infiltrates or lymphomas are not likely to be seen in skin, but mediastinal masses may be seen. A proliferation of neuroblasts occurs in neuroblastoma, a common childhood neoplasm in the abdomen.

A 3-year-old child's mother notes that a large port wine stain on the left side of the child's face has not diminished in size since birth. This irregular, slightly raised, red-blue area is not painful, but is very disfiguring. Histologically, this lesion is most likely composed of a proliferation of which of the fol- lowing tissue components? A Capillaries B Fibroblasts C Lymphatics D Lymphoblasts E Neuroblasts

E This patient has clinical features of neurofibromatosis type 1. The NF1 gene encodes for a GTPase-activating protein that facilitates the conversion of active (GTP-bound) RAS to inactive (GDP-bound) RAS. Loss of NF1 prevents such con- version and traps RAS in the active signal-transmitting stage that drives cell proliferation. Thus, the wild type NF1 gene acts as a tumor suppressor. All other listed mechanisms also are involved in carcinogenesis, but in different tumor types.

A 30-year-old man has a 15-year history of increasing numbers of benign skin nodules. On physical examination, the firm, nontender, subcutaneous nodules average 0.5 to 1 cm. Further examination shows numerous oval 1- to 5-cm flat, light brown skin macules. Ophthalmoscopic examination shows hamartomatous nodules on the iris. A biopsy specimen of one skin nodule shows that it is attached to a peripheral nerve. Which of the following molecular abnormalities is most likely related to his clinical presentation? A Decreased susceptibility to apoptosis B Impaired functioning of mismatch repair C Increased production of epidermal growth factor D Lack of nucleotide excision repair E Persistent activation of the RAS gene F Reduced expression of RB protein

G Angiogenesis is a key feature of neoplasms because the growing tumor needs a blood supply, and up-regulation of factors such as VEGF and FGF help to keep the cancer growing. VEGF may be up-regulated by activation of hypoxia-inducible factor 1-alpha (HIF-1-alpha). The von Hippel-Lindau (VHL) gene acts as a tumor suppressor, and it normally produces a protein that binds to hypoxia-inducible factor 1-alpha so that it is cleared. VHL mutation leads to loss of this binding protein and activation of angiogenesis factors. Individuals with VHL syndrome have various neoplasms, including pheochromocytomas, renal cell carcinomas, and hemangioblastomas. The other listed genes have products that do not directly act on angiogenesis pathways.

A 30-year-old man has a pheochromocytoma of the left adrenal gland; a sibling had a cerebellar hemangioblastoma. He undergoes adrenalectomy, and on microscopic examina- tion there is extensive vascularity of the neoplasm. Mutational analysis of the neoplastic cells shows that both allelic cop- ies of a gene have been lost, so that a protein that binds to hypoxia-inducible factor 1-alpha is no longer ubiquitinated, but instead translocates to the nucleus and activates transcrip- tion of VEGF. Which of the following genes is most likely mutated in this man? A APC B BCL2 C EGF D HER2 E HST1 F MYC G VHL

The answer is C: Pyknosis. Coagulative necrosis refers to light microscopic alterations in dying cells. When stained with the usual combination of hematoxylin and eosin, the cytoplasm of a necrotic cell is eosinophilic. The nucleus displays an initial clumping of chromatin followed by its redistribution along the nuclear membrane. In pyknosis, the nucleus becomes smaller and stains deeply basophilic as chromatin clumping continues. Karyorrhexis (choice B) and karyolysis (choice A) represent further steps in the fragmentation and dissolution of the nucleus. These steps are not evident in the necrotic cells shown in this Pap smear.

A 31-year-old woman complains of increased vaginal discharge of 1-month duration. A cervical Pap smear is shown in the image. Superficial epithelial cells are identified with arrows. When compared to cells from the deeper intermediate layer (top), the nuclei of these superfi cial cells exhibit which of the following cytologic features? (A) Karyolysis (B) Karyorrhexis (C) Pyknosis (D) Segmentation (E) Viral inclusion bodies

G A teratoma is a neoplasm derived from totipotential germ cells that differentiate into tissues that represent all three germ layers: ectoderm, endoderm, and mesoderm. When the elements all are well differentiated, the neoplasm is "mature" (benign). Adenocarcinomas have malignant- appearing glandular elements. Fibroadenomas have a benign glandular and stromal component; they are common in the breast. Gliomas are found in the central nervous system. Hamartomas contain a mixture of cell types common to a tissue site; the lung is one site for this uncommon lesion. A mesothelioma arises from the lining of thoracic and abdominal body cavities. A rhabdomyosarcoma comprises cells that poorly resemble striated muscle; most arise in soft tissues.

A 32-year-old woman has experienced dull pelvic pain for the past 2 months. Physical examination shows a right ad- nexal mass. An abdominal ultrasound scan shows a 7.5-cm cystic ovarian mass. The mass is surgically excised. The sur- face of the mass is smooth, and it is not adherent to surround- ing pelvic structures. On gross examination, the cystic mass is filled with hair. Microscopically, squamous epithelium, tall columnar glandular epithelium, cartilage, and fibrous connec- tive tissue are present and resemble normal tissue counter- parts. Which of the following is the most likely diagnosis? A Adenocarcinoma B Fibroadenoma C Glioma D Hamartoma E Mesothelioma F Rhabdomyosarcoma G Teratoma

C Recurrent fetal loss suggests a parental cause. A chromosomal abnormality such as a Robertsonian trans- location may account for stillbirths, particularly in the first trimester, when many fetal losses result from chromosomal abnormalities. A genome-wide associations study applies to populations, not individuals, and establishes linkages of common diseases such as hypertension and diabetes mel- litus to polygenic risks. Fluorescence in situ hybridization (FISH) analysis is useful for establishing chromosome num- ber and morphology, such as translocations, inversions, and deletions, but is not as definitive as a karyotype. PCR analy- sis is useful for identifying specific gene defects, not whole chromosomal abnormalities. Tandem mass spectroscopy is used in newborn screening for biochemical inborn errors of metabolism, such as phenylketonuria.

A 32-year-old woman has had three pregnancies, all ending in stillbirths in the first trimester. On physical exami- nation, she and her only spouse for all pregnancies have no abnormalities. Which of the following laboratory tests is most appropriate to perform on this woman for elucidating poten- tial causes for recurrent fetal loss? A Genome-wide association study B Fluorescence in situ hybridization C Karyotyping D PCR analysis E Tandem mass spectroscopy

The answer is C: Hyperplasia. Infants of diabetic mothers show a 5% to 10% incidence of major developmental abnormalities, including anomalies of the heart and great vessels and neural tube defects. The frequency of these lesions relates to the control of maternal diabetes during early gestation. During fetal development, the islet cells of the pancreas have proliferative capacity and respond to increased demand for insulin by undergoing physiologic hyperplasia. Fetuses exposed to hyperglycemia in utero may develop hyperplasia of the pancreatic β cells, which may secrete insulin autonomously and cause hypoglycemia at birth. Metaplasia (choice D) is defined as the conversion of one differentiated cell pathway to another.

A 32-year-old woman with poorly controlled diabetes mellitus delivers a healthy boy at 38 weeks of gestation. As a result of maternal hyperglycemia during pregnancy, pancreatic islets in the neonate would be expected to show which of the follow- ing morphologic responses to injury? (A) Atrophy (B) Dysplasia (C) Hyperplasia (D) Metaplasia (E) Necrosis

A Inflammation from reflux of gastric acid has resulted in replacement of normal esophageal squamous epithelium by intestinal-type columnar epithelium with goblet cells. Such conversion of one adult cell type to another cell type is called metaplasia, and it occurs when stimuli reprogram stem cells. Goblet cells are not normal constituents of the esophageal mucosa, and they are a minor part of this meta- plastic process. The lamina propria has some inflammatory cells, but it does not atrophy. The squamous epithelium does not become dysplastic from acid reflux, but the columnar metaplasia may progress to dysplasia, not seen here, if the abnormal stimuli continue. These cells are not significantly increased in size (hypertrophic).

A 34-year-old obese woman has experienced heartburn from gastric reflux for the past 5 years after eating large meals. She undergoes upper gastrointestinal endoscopy, and a biopsy specimen of the distal esophagus is obtained. Which of the following microscopic changes, seen in the figure, has most likely occurred? A Columnar metaplasia B Goblet cell hyperplasia C Lamina propria atrophy D Squamous dysplasia E Mucosal hypertrophy

E. Infection by HHV8, also called Kaposi sarcoma herpesvirus (KSHV), is associated with Kaposi sarcoma (KS) and can be acquired as a sexually transmitted disease. KS is a complication of AIDS. Individuals with HIV infection can be infected with various viruses, including cytomegalovirus (CMV) and Epstein-Barr virus (EBV), but these viruses have no etiologic association with KS. EBV is a factor in the development of non-Hodgkin lymphoma (Burkitt's lymphoma), and CMV can cause pneumonitis or retinitis, or it can be disseminated. Hyperlipidemia may complicate antiretroviral therapy, but does not produce skin nodules. MAC infection can be seen in HIV-infected patients as well, but it produces small granulomas, mainly in tissues of the mononuclear phagocyte system.

A 35-year-old man is known to have been HIV-positive for the past 10 years. Physical examination now shows multiple reddish purple, nodular skin lesions. These lesions have been slowly increasing for the past year. Which of the following risk factors is most likely to play a role in the development of these skin lesions? A. Antiretroviral therapy B. Epstein-Barr virus infection C. hyperlipidemia D. Mycobacterium avium complex infection E. Sexual intercourse

B

A 36 year old male bodybuilder was referred for management of persistent atrial fibrillation (AF, an irregular and often very fast heart rate) of uncertain duration. He had been aware of irregular palpitations over the preceding 6 months which were associated with significant fatigue but without syncope (temporary loss of consciousness caused by a fall in blood pressure), dyspnea (difficult or labored breathing), or chest pain. Further questioning revealed that the onset of palpitations had coincided with the patient's commencement on illicitly obtained anabolic-androgenic steroids. (i) What are the other possible side effects of the anabolic-androgenic steroid abuse in this patient? (ii) If this patient were a female, what side effects are expected? Choose ONE CORRECT pair of answers in the following. A. (i) Gynecomastia; (ii) Breast hypertrophy (macromastia). B. (i) Testicular atrophy (hypogonadism); (ii) Hirsutism (excessive body hair). C. (i) Testicular hypertrophy (hypergonadism); (ii) Clitoral hypertrophy. D. (i) Deepening of voice; (ii) Breast hypertrophy (macromastia). E. (i) Breast tissue atrophy; (ii) Acne development.

E This patient has AIDS, with plaque like to nodular reddish purple lesions. Kaposi sarcoma is associated with a herpesvirus agent that is sexually transmitted: human her- pesvirus 8 (HHV-8), also called the Kaposi sarcoma herpesvirus. Other herpesviruses are not involved in the pathogenesis of Kaposi sarcoma, although infection with these viruses can occur frequently in individuals with AIDS. HIV, although present in the lymphocytes and monocytes, is not detected in the spindle cells that proliferate in Kaposi sarcoma. With the exception of varicella-zoster virus, which is associated with dermatomally distributed skin vesicles known as shingles, skin lesions are uncommon manifestations of herpesviruses, which include cytomegalovirus, Epstein-Barr virus (EBV), and adenovirus.

A 37-year-old man who is HIV-positive has noticed an in- creasing number and size of skin lesions on his face, trunk, and extremities, as shown in the figure, over the past 18 months. Some of the larger lesions appear to be nodular. Molecular analysis of the spindle cells found in these skin lesions is likely to reveal the genome of which of the following viruses? A Adenovirus B Cytomegalovirus C Epstein-Barr virus D HIV-1 E Human herpesvirus-8

C The MYC oncogene is commonly activated in Burkitt lymphoma because of a t(8;14) translocation. The MYC gene binds DNA to cause transcriptional activation of growth- related genes such as that for cyclin D1, resulting in activa- tion of the cell cycle. EGF (such as HER2 in breast cancers) encodes the epithelial growth factor receptor located on the cell surface. p53 and APC are tumor suppressor genes that are inactivated in many cancers, including colon cancer. RAS oncogene encodes a GTP-binding protein that is located un- der the cell membrane.

A 38-year-old woman has abdominal distention that has been worsening for the past 6 weeks. An abdominal CT scan shows bowel obstruction caused by a 6-cm mass in the jejunum. At laparotomy, a portion of the small bowel is re- sected. Flow cytometric analysis of a portion of the tumor shows a clonal population of B lymphocytes with high S phase. Translocation with activation of which of the follow- ing nuclear oncogenes is most likely to be present in this tumor? A APC B EGF C MYC D p53 E RAS

A The many focal, chalky white deposits in the mesentery, composed mainly of adipocytes, are areas of fat necrosis. The deposits result from the release of pancreatic enzymes such as lipases in a patient with acute pancreatitis. Gangrenous necrosis is mainly coagulative necrosis, but occurs over an extensive area of tissues. Viral hepatitis does not cause cell necrosis in organs other than liver, and hepatocyte necrosis from viral infections occurs mainly by means of apoptosis. Intestinal infarction is a form of coagulative necrosis. Infection with tuberculosis leads to caseous necrosis.

A 38-year-old woman has experienced severe abdominal pain over the past day. On examination she is hypotensive and in shock. Laboratory studies show elevated serum lipase. From the representative gross appearance of the mesentery shown in the figure, which of the following events has most likely occurred? A Acute pancreatitis B Gangrenous cholecystitis C Hepatitis B virus infection D Small intestinal infarction E Tuberculous lymphadenitis

E Cushing's syndrome is a disease caused by the dysregulation of the Hypothalamus-Pituitary-Adrenal axis. This case is an example of ACTH-independent Cushing's syndrome. A gene coding for the catalytic domain of protein kinase A is often mutated in the adrenal tumors, resulting in a constitutively active PKA that signals to produce cortisol. Other mutations that occur in the ACTH-triggered GPCR signaling pathway can also result in similar disease. There are other types of Cushing's syndrome: Cushing disease (a.k.a. ACTH-dependent Cushing's syndrome resulting from pituitary tumors) or ectopic Cushing syndrome caused by tumors making ACTH ectopically.

A 38-year-old woman presented with "moon face," "buffalo hump," and weight gain of 9 kg in 12 months. The 24-hour urine cortisol level was elevated to 101 μg/day (normal range 0 - 50). The initial contrast-enhanced abdominal computed tomography (CT) showed a 2.8-cm left adrenal mass, which was further enhanced in arterial phase CT. Laboratory studies show markedly decreased ACTH (adrenocorticotropic hormone). Neither low- nor high-dose dexamethasone (cortisol agonist) suppresses the patient's cortisol production. What kind of gene mutation might have occurred in the cells of the adrenal mass of this patient? A. A mutation in a gene coding for Gαs, that inhibits GDP/GTP exchange. B. A mutation in a gene coding for GαI, that inhibits GDP/GTP exchange. C. A mutation in a gene coding for Gαq, that inhibits GDP/GTP exchange. D. A mutation in a gene coding for cAMP phosphodiesterase, that activates the enzyme constitutively. E. A mutation in a gene coding for protein kinase A, that makes the enzyme constitutively active.

E A is wrong since the mutation in the intracellular domain is not likely to affect the hormone binding. The mutation will decrease the activity of PI3K, glucose uptake by GLUT 4 transportes. It will not activate the glycogen synthesis since Akt activity will be decreased by the mutation.

A 40 year old, overweight male patient, presents to you with high blood glucose, increased thirst, frequent urination, weight loss, fatigue, frequent infections and blurred vision. His DNA was extracted from his peripheral blood and the gene coding for the insulin receptor was amplified by polymerase chain reaction. Sequencing of the resulting DNA reveals a mutation in the intracellular domain that decreases its catalytic activity. Which one of the following statements describes the most likely result of the mutation? A. Decreased affinity of binding of insulin to the insulin receptor. B. Increased activity of phosphoinositide 3-kinase (PI3K). C. Increased glucose uptake through GLUT 4 transporters. D. Increased activity of glycogen synthesis. E. Decreased activity of Akt (protein kinase B)

A A discrete small mass such as that described is probably benign. Adenomas arise from epithelial surfaces. Though adenocarcinoma may arise from a colonic adenoma, such malignant lesions tend to be larger and more irregular. A choristoma is a benign mass composed of tissues not found at the site of origin. A hamartoma is a rare benign mass composed of tissues usually found at the site of origin. A hyperplastic colonic lesion tends to be smaller and flatter. A sarcoma is a malignant neoplasm arising in mesenchymal tissues, not in epithelium.

A 40-year-old man has a positive stool guaiac test during a routine physical examination. A colonoscopy is performed and a 0.9-cm, circumscribed, pedunculated mass on a short stalk is found in the upper rectum. Which of the following terms best describes this lesion? A Adenoma B Carcinoma C Choristoma D Hamartoma E Hyperplasia F Sarcoma

E Worldwide, increasing numbers of skin cancers occur because of sun exposure. The ultraviolet light damages the skin and damages cellular DNA, leading to mutations that can escape cellular repair mechanisms. Allergic reactions do not promote cancer. Asbestos exposure increases lung carci- noma risk in smokers and can lead to rare mesotheliomas of pleura. Chemotherapeutic agents have carcinogenic poten- tial, particularly alkylating agents such as cyclophosphamide, but leukemias and lymphomas are the usual result. Smoking tobacco is related to many cancers, but skin cancers are not typically associated with this risk factor.

A 42-year-old man is concerned about a darkly pigment- ed "mole" on the back of his hand. The lesion has enlarged and bled during the past month. On physical examination, there is a slightly raised, darkly pigmented, 1.2-cm lesion on the dorsum of the right hand. The lesion is completely excised. The microscopic appearance is shown in the figure. Which of the following factors presents the greatest risk for the develop- ment of this neoplasm? A Allergy to latex B Asbestos exposure C Chemotherapy D Smoking tobacco E Ultraviolet radiation

D. After a hydatidiform mole, it is very important to assure that all the invasive trophoblastic tissue is removed. High levels of hCG are a good indicator of retained trophoblastic tissue because such tissue produces this hormone. In this case, the trophoblastic tissue has developed into a malignant choriocarcinoma and metastasized to the brain, causing her symptoms of headache, blurred vision, and so on.

A 42-year-old woman presents with complaints of severe headaches, blurred vision, slurred speech, and loss of muscle coordination. Her last pregnancy 5 years ago resulted in a hydatidiform mole. Laboratory results show a high hCG level. Which of the following conditions is a probable diagnosis? (A) Vasa previa (B) Placenta previa (C) Succenturiate placenta (D) Choriocarcinoma (E) Membranous placenta

D When Ras G-protein loses its GTPase activity, it can be constitutively activated.

A 43-year-old female was diagnosed with adenoid cystic carcinoma (AdCC), one of the most common salivary gland carcinomas. Sequencing of the biopsy revealed a mutation in a RAS gene, resulting in a constitutively activated Ras protein. What function of the protein would have been inhibited? A. ATP (adenosine triphosphate) hydrolysis activity. B. CTP (cytidine triphosphate ) hydrolysis activity. C. cAMP (cyclic adenosine monophosphate) hydrolysis activity. D. GTP (guanosine triphosphate) hydrolysis activity. E. cGMP (cyclic guanosine monophosphate ) synthesis activity.

B The figure shows a pale ischemic infarction of the renal cortex extending nearly to the renal capsule, a typi- cal finding when a medium-sized arterial thromboembolus lodges in a peripheral renal artery branch. The infarct is wedge-shaped, typical for many parenchymal organs, because there is minimal collateral circulation. An abscess is a form of liquefactive necrosis from a localized collection of neutrophils in association with infection, and though it could be yellowish, it is likely to be round. Liquefactive necrosis from arterial occlusion and infarction occurs in the brain. Multiorgan failure occurs with shock, and whole organs are affected by ischemia. Venous thrombosis tends to produce hemorrhagic lesions.

A 44-year-old man with dilated cardiomyopathy and heart failure develops left atrial mural thrombosis. He develops the complication shown in the figure, manifested by hematuria. Which of the following is the best term for this complication? A Abscess B Ischemic infarct C Liquefactive necrosis D Multiorgan failure E Venous thrombosis

C Lymphatic spread, especially to regional lymph nodes draining from the primary site, is typical of a carcinoma. An intraductal carcinoma has not extended beyond the basement membrane, but an infiltrating carcinoma has acquired the ability to invade and spread via metastasis. The primary site may be difficult to detect if small or deep, and hence the need for radiologic imaging, such as mammography. A fibroadenoma is a benign neoplasm and cannot invade or metastasize. Infection from a breast abscess can spread to the lymph nodes, but the resulting nodal enlargement is typically associated with pain—a cardinal sign of acute inflammation. Sarcomas uncommonly metastasize to lymph nodes, and a leiomyosarcoma of breast is rare.

A 44-year-old woman feels painless lumps in her armpit, which were not present a month ago. On examination, right axillary lymphadenopathy is present. The nodes are painless but firm. Which of the following is the most likely lesion in her right breast? A Acute mastitis with abscess B Fibroadenoma C Infiltrating lobular carcinoma D Intraductal carcinoma E Leiomyosarcoma

B

A 45-year-old woman is investigated for hypertension and is found to have enlargement of the left kidney. The right kidney is smaller than normal. Contrast studies reveal stenosis of the right renal artery. The size change in the right kidney is an example of which of the following adaptive changes? (A) Aplasia (B) Atrophy (C) Hyperplasia (D) Hypertrophy (E) Metaplasia

A The appearance of lipid vacuoles in many of the hepatocytes is characteristic of fatty change (steatosis) of the liver. Abnormalities in lipoprotein metabolism can lead to steatosis. Alcohol is a hepatotoxin acting via increased acetaldehyde accumulation that promotes hepatic steatosis. Decreased serum albumin levels and increased prothrombin time suggest alcohol-induced hepatocyte damage. Aspirin has a significant effect on platelet function, but not on hepatocytes. Substance abuse with heroin produces few organ specific pathologic findings. Exercise has little direct effect on hepatic function. Smoking directly damages lung tissue, but has no direct effect on the liver.

A 46-year-old man has noted increasing abdominal size for the past 6 years. On physical examination his liver span is increased to 18 cm. An abdominal CT scan shows an enlarged liver with diffusely decreased attenuation. Laboratory findings include increased total serum cholesterol and triglyceride levels, increased prothrombin time, and a decreased serum albumin concentration. The representative microscopic appearance of his liver is shown in the figure. Which of the following activities most likely led to these findings? A Drinking beer B Ingesting aspirin C Injecting heroin D Playing basketball E Smoking cigarettes

C Several pathologic mechanisms play a role in the development of tumor metastases. The tumor cells first must become discohesive and detach from the primary site, degrade the basement membrane and interstitial connective tissue, and then attach elsewhere to become metastases. Reduced expression of adhesion molecules such as E-cadherins promotes metastases. Tumor cells can elaborate, not reduce, proteases such as metalloproteinases to promote discohesiveness. Expression of estrogen and progesterone receptors predicts breast cancer responsiveness to antihormone therapy, and there is a monoclonal antibody, trastuzumab, that targets HER-2, a form of epidermal growth factor receptor. Tyrosine kinase receptors within cells aid in signaling cell growth.

A 48-year-old woman notices a lump in her left breast. On physical examination she has a firm, nonmovable, 2-cm mass in the upper outer quadrant of the left breast. There are enlarged, firm, nontender lymph nodes in the left axilla. A fine-needle aspiration biopsy is performed, and the cells pres- ent are consistent with carcinoma. A lumpectomy with axillary lymph node dissection is performed, and carcinoma is present in two of eight axillary nodes. Reduced expression of which of the following molecules by the tumor cells is most likely responsible for the lymph node metastases? A Estrogen receptors B ERBB2 (HER-2) C E-cadherin D Progesterone receptors E Tyrosine kinases

D Growth hormone triggers the JAK-STAT pathway signaling.

A 5-year old boy was brought into your clinic because his mother was concerned that he was the smallest kid in his class and was no larger than his 3-year old brother. When measured, he was found to be in the lower 30th percentile of his age group. Neither parent was extremely small. Serum levels of insulin were in the normal range. He was given an insulin tolerance test** and his resultant growth hormone levels were found to be well below normal (normal is >30 IU/L). Serum levels of insulin like growth factor-1 (IGF-1) were determined and were very low. Growth hormone injection treatment was prescribed for the hormone deficiency. **In insulin tolerance test, insulin is given to lower the blood glucose. This signals the pituitary gland to release two hormones: the growth hormone (GH) and adrenocorticotropic hormone (ACTH). In turn, ACTH signals the adrenal cortex to release cortisol. At intervals during the test, blood samples are taken to measure the growth hormone and cortisol. Which one of the following signaling pathways would be directly activated by the binding of the administered hormone to its receptor? A. cAMP pathway. B. phosphoinositide pathway (DAG and IP3/Ca2+). C. Ras-MAPK pathway. D. JAK-STAT pathway. E. cGMP pathway.

D This patient has a classic history and t(9;22) translocation with chronic myelogenous leukemia. The translocation causes uncontrolled nonreceptor tyrosine kinase activity of the BCR-ABL fusion gene. These patients undergo remission with drugs such as imatinib that inhibit tyrosine kinases. Agents that activate caspases theoretically may help in many cases, especially when apoptosis is blocked as in tumors with BCL2 overexpression. Antibodies to epithelial growth factor receptors, such as ERBB2 (HER2) receptors, are beneficial in certain breast tumors with amplification of this gene. Delivery of p53 into cells by viral vectors has not yet been proven to be valuable in cancer treatment, and it is not used in chronic myelogenous leukemia. Translocation of β-catenin to the nucleus occurs in colon cancers when there is mutational loss of APC genes.

A 50-year-old woman has had easy fatigability and noted a dragging sensation in her abdomen for the past 5 months. Physical examination reveals that she is afebrile. She has marked splenomegaly, but no lymphadenopathy. Laboratory studies show her total WBC count is 250,000/mm3 with WBC differential count showing 64% segmented neutrophils, 11% band neutrophils, 7% metamyelocytes, 5% myelocytes, 4% myeloblasts, 3% lymphocytes, 2% basophils, 2% eosinophils, and 2% monocytes. A bone marrow biopsy is performed, and karyotypic analysis of the cells reveals a t(9;22) translocation. Medical treatment with a drug having which of the following modes of action is most likely to produce a complete remission in this patient? A Activating cellular caspases B Antibody binding to EGF receptors C Delivering normal p53 into cells with viral vectors D Inhibiting tyrosine kinase activity E Preventing translocation of β-catenin to the nucleus

D A true neoplasm is a monoclonal proliferation of cells, whereas a reactive proliferation of cells is not monoclonal. Molecular genetic analysis, such as allelotype analysis with microsatellite markers, shows clonality. Reactive and neoplastic cellular proliferations may have similar histochemical and immunohistochemical staining patterns based on the type of cells and proteins that are present. Flow cytometry is effective at indicating the DNA content, aneuploidy, and growth frac- tion, but does not indicate clonality.

A 50-year-old woman undergoes screening colonoscopy as part of a routine health maintenance work-up. An isolated 1-cm pedunculated polyp is found in the sigmoid colon. The excised polyp histologically shows well-differentiated glands with no invasion of the stalk. Which of the following investigational research procedures can distinguish most clearly whether the polyp represents hyperplasia of the colonic mucosa or a tubular adenoma? A Flow cytometry to quantitate cells in the S phase B Histochemical staining for mucin C Immunohistochemical staining for keratin D Molecular marker of clonality

A

A 50-yr old man is diagnosed with mitral valve dysfunction. He undergoes surgery to implant an artificial valve. Following surgery, he is prescribed daily anticoagulant (warfarin, 5 mg/day, half life ~48h, 97% binding to plasma proteins) for the rest of his life, to prevent blood clotting. He is also prescribed an antibiotic (ciprofloxacin, 500 mg/day, half life ~5h, 90% binding to plasma proteins) for two weeks, to prevent infections. This warfarin+ciprofloxacin treatment reduces blood clotting for this patient with an artificial mitral valve to an "optimal" level (note, there is no "optimal level", however this treatment produces low level of clotting without producing any excessive bleeding). Which adverse effect(s) is (are) most likely to occur after withdrawing ciprofloxacin treatment, while continuing warfarin treatment at the same dose of 5mg/day? A. blood clotting B. blood clotting for the first day, then excessive bleeding C. blood clotting for the first 8-10 days, then excessive bleeding D. excessive bleeding E. excessive bleeding for the first 8-10 days, then blood clotting

F Vinyl chloride is a rare cause of liver cancer. This causal relationship was easy to show, however, because hepatic angiosarcoma is a rare neoplasm. Arsenic is a risk factor for skin cancer. Asbestos exposure is linked to pleural malignant mesothelioma and to bronchogenic carcinomas in smokers. Benzene exposure is linked to leukemias. Beryllium exposure can produce interstitial lung disease and lung cancer. Nickel exposure increases the risk of respiratory tract cancers. Expo- sure to naphthalene compounds is a risk factor for cancers of the urinary tract.

A 51-year-old man who works in a factory that produces plastic pipe has experienced weight loss, nausea, and vomit- ing over the past 4 months. On physical examination, he has tenderness to palpation in the right upper quadrant of the ab- domen, and the liver span is increased. Laboratory findings include serum alkaline phosphatase, 405 U/L; AST, 45 U/L; ALT, 30 U/L; and total bilirubin, 0.9 mg/dL. An abdominal CT scan shows a 12-cm mass in the right lobe of the liver. A liv- er biopsy is performed, and microscopic examination shows a malignant neoplasm of endothelial cells. The patient has most likely been exposed to which of the following agents? A Arsenic B Asbestos C Benzene D Beryllium E Nickel F Vinyl chloride G Naphthalene

A The PET scan is based upon selective uptake of a glu- cose derivative into tumor cells. The Warburg effect occurs when cancer cells shift their metabolism to aerobic glycolysis for selective growth advantage under harsh circumstances. Glycolysis also yields pyruvate for anabolic demands of in- creased tumor doublings. The p53 and c-MYC genes favor this metabolic change, whereas PTEN inhibits tumor cell autophagy, giving cancer cells an edge in growth. Cancer cells are less differentiated than normal cells and thus have decreased ability to do many complex biochemical processes, so they favor a simple one—glycolysis. Gluconeogenesis is a function of hepatocytes in response to decreased caloric intake. The HMP shunt and Krebs cycle are more useful to normal cells maintaining themselves at the status quo. Neo- plasms generate large amounts of purines from cell divisions and cell turnover that must be eliminated as uric acid, but neoplastic cells do not perform this task.

A 53-year-old man diagnosed with oral cancer and treated with radiation and chemotherapy 1 year ago now has a positron emission tomography (PET) scan of his neck that shows a single focus of increased uptake. This focus is resected and microscopic examination shows that it is a metastasis. Mo- lecular analysis of this cancer shows p53, PTEN, and c-MYC gene mutations. Which of the following metabolic pathways is most likely up-regulated to promote his cancer cell survival and proliferation? A Aerobic glycolysis B Gluconeogenesis C Hexose monophosphate shunt D Oxidative phosphorylation E Purine degradation

F. The pressure load on the left ventricle results in an increase in myofilaments in the existing myofibers, so they enlarge. The result of continued stress from hypertension is eventual heart failure with decreased contractility. Apoptosis would lead to loss of cells and diminished size. Dysplasia is not a diagnosis made for the heart. Hemosiderin deposition in the heart is a pathologic process resulting from increased iron stores in the body. Though hyperplasia from proliferation of myofibroblasts is possible, this does not contribute significantly to cardiac size. Metaplasia of muscle does not occur, although loss of muscle occurs with aging and ischemia as myofibers are replaced by fibrous tissue.

A 53-year-old woman with no prior illnesses has a routine checkup by her physician. On examination, she has a blood pressure of 150/95 mm Hg. If her hypertension remains untreated for years, which of the following cellular alterations would most likely be seen in her myocardium? A. apoptosis B. dysplasia C. fatty change D. hemosiderosis E. hyperplasia F. hypertrophy G. metaplasia

E If existing cell damage is not great after myocardial infarction, the restoration of blood flow can help prevent fur- ther cellular damage. However, the reperfusion of damaged cells results in generation of oxygen-derived free radicals, causing a reperfusion injury. The elevation in the CK level is indicative of myocardial cell necrosis, because this intracel- lular enzyme does not leak in large quantities from intact myocardial cells. Myocardial fibers do not regenerate to a significant degree, and atrophic fibers would have less CK to release. t-PA does not produce a toxic chemical injury; it induces thrombolysis to restore blood flow in occluded coronary arteries.

A 54-year-old man experienced severe substernal chest pain for 3 hours. An ECG showed changes consistent with an acute myocardial infarction. After thrombolytic therapy with tissue plasminogen activator (t-PA), his serum creatine kinase (CK) level increased. Which of the following tissue events most likely occurred in the myocardium after t-PA therapy? A Cellular regeneration B Drug toxicity C Increased synthesis of CK D Myofiber atrophy E Reperfusion injury

A Infiltrating ductal and intraductal carcinoma are present in the figure. Increased expression of ERBB2 (HER2) can be detected immunohistochemically and by fluorescence in situ hybridization (FISH) in the biopsy specimen. One third of breast cancers may show this change. Such amplification is associated with a poorer prognosis. Detection of a specific gene product in the tissue has value for determination of treatment and prognosis. BRCA1 and p53 mutations, if inherited in the germ line, can predispose the patient to breast cancer and other tumors. With BRCA1, there is family history of breast cancer, often at a young age. The tumor suppressor gene p53 mutations predispose to many types of cancers. An inherited deletion of RB gene predisposes to retinoblastoma. The BCR-ABL fusion product, seen in chronic myeloid leukemia, often results from t(9;22).

A 54-year-old woman notes a lump in her right breast. Physical examination shows a 2-cm mass fixed to the under- lying tissues beneath the areola and three firm, nontender, lymph nodes palpable in the right axilla. There is no family history of cancer. An excisional breast biopsy is performed, and microscopic examination shows the findings in the fig- ure. Over the next 6 months, additional lymph nodes become enlarged, and CT scans show nodules in the lung, liver, and brain. Which of the following molecular abnormalities is most likely to be found in her carcinoma cells? A Amplification of the ERBB2 (HER2) gene B Deletion of one RB gene copy C Fusion of BCR and C-ABL genes D Inactivation of one BRCA1 gene copy E Mutation of one p53 gene copy

E This is the definition of desmoplasia.

A 54-year-old woman who has been diagnosed with early stage breast cancer undergoes surgery for a lumpectomy to remove a small tumor detected by mammography. The pathology report confirms the early stage of the cancer and further comments on the fact that there is significant desmoplasia in the surrounding tissue. The term desmoplasia refers to (A) an irregular accumulation of blood vessels. (B) maturation and spatial arrangement of cells. (C) metastatic involvement of surrounding tissue. (D) normal tissue misplaced within another organ. (E) proliferation of non-neoplastic fibrous connective tissue.

C A p53 mutation involving both wild type alleles is one of the most common genetic alterations in human can- cers, including the most common cancers—lung, colon, and breast. The loss of this tumor suppressor activity indicates that the cell cycle is not properly arrested in the late G1 phase, and when DNA damage occurs, DNA repair cannot be com- pleted before the cell proliferates. Inability to hydrolyze GTP is a result of RAS oncogene activation. Growth factors such as EGF are activators of the cell cycle to promote cell growth. Microsatellite instability occurs with mutation in genes, such as hMSH2, that repair DNA damage. Transcriptional activa- tion is a feature of the MYC proto-oncogene.

A 55-year-old man has had hemoptysis and worsening cough for the past month. On physical examination, wheezes are auscultated over the right lung posteriorly. A chest radiograph shows a 6-cm right perihilar mass. A fine-needle aspiration biopsy is performed and yields cells with the microscopic appearance of non-small cell bronchogenic carcinoma. Molecular analysis of the neoplastic cells shows a p53 gene mutation. Which of the following mechanisms has most likely produced the neoplastic transformation? A Inability to hydrolyze GTP B Growth factor receptor activation C Loss of cell cycle arrest D Microsatellite instability E Transcriptional activation

E There is increasing evidence that localization of cancer metastases is influenced by the expression of chemo- kine receptors by cancer cells and elaboration of their ligands (chemokines) by certain tissues. In the case of breast cancer, the carcinoma cells express CXCR4 chemokines. Vascular, lymphatic, or basement membrane invasion is required for metastases, but these characteristics do not dictate accurately the location of metastases.

A 55-year-old woman has felt an enlarging lump in her left breast for the past year. A hard, irregular 5-cm mass fixed to the underlying chest wall is palpable in her left breast. Left axillary nontender lymphadenopathy is noted. There is no hepatosplenomegaly. A chest CT scan reveals multiple bilateral pulmonary "cannonball" nodules. A left breast bi- opsy is performed, and on microscopic examination shows high-grade infiltrating ductal carcinoma. The appearance of the nodules in her lungs is most likely related to which of the following? A Internal mammary artery invasion by carcinoma cells B Lymphatic connections between the breast and the pleura C Overexpression of estrogen receptors within the carcinoma cell nuclei D Proximity of the breast carcinoma to the lungs E Pulmonary chemokines that bind carcinoma cell chemokine receptors

D Estrogen, similar to many other hormones and drugs, by itself is not carcinogenic, but it is responsible for stimulation of endometrial growth (hyperplasia), which has a promoting effect when cellular mutations occur to produce carcinoma. Inherited susceptibility can never be completely excluded when an individual has two tumors; this can occur in patients with inherited mutations in the p53 gene. In this case, however, there is a clear hormonal basis for the second tumor. Faulty tumor suppressor genes are not involved in hormonal promotion of a neoplasm. A paraneoplastic syndrome results from ectopic secretion of a hormone by tumor (e.g., lung cancer cells producing corticotropin). Tumor heterogeneity does not refer to two separate kinds of neoplasms; it refers to heterogeneity with a given tumor or metastasis.

A 56-year-old woman has had vaginal bleeding for 1 week. Her last menstrual period was 10 years ago. On physical examination, a lower abdominal mass is palpated. An endometrial biopsy is performed and shows endometrial carcinoma. An abdominal CT scan shows a 6-cm mass in the left ovary. A total abdominal hysterectomy is performed. Microscopically, the ovarian mass is a granulosa-theca cell tu- mor producing estrogen. Which of the following best describes the relationship between these two neoplasms? A Genetic susceptibility to tumorigenesis B Mutational inactivation of a tumor suppressor gene C Paraneoplastic syndrome D Promotion of carcinogenesis E Tumor heterogeneity

A The cells shown in the figure show marked pleomorphism and hyperchromatism (anaplasia), and it is difficult to discern the cell of origin because no differentiation is noted. A bizarre tripolar mitotic figure is present. This degree of anaplasia is consistent with an aggressive, high-grade malignancy called anaplastic carcinoma. Apoptosis is single cell necrosis, but the cells shown appear viable and not fragmented. Dysplasia refers to changes within an epithelium that presage a neoplasm. Metaplasia with one epithelial cell type substituted for another may presage dysplasia and malignancy. Well-differentiated neoplasms tend to be less aggressive and slower growing and resemble the cell of origin.

A 57-year-old woman has experienced an increasing feeling of fullness in her neck along with a 3-kg (7-lb) weight loss over the past 3 months. On physical examination, there is a firm, fixed mass in a 3 × 5 cm area in the right side of the neck. A CT scan shows a solid infiltrating mass in the region of the right lobe of the thyroid gland. A biopsy of the mass is performed and the microscopic appearance is shown in the figure. All areas of the tumor have similar morphology. Which of the following terms best describes this neoplasm? A Anaplastic B Apoptotic C Dysplastic D Metaplastic E Well-differentiated

E. The markedly thickened left ventricular wall is characteristic of myocardial fiber hypertrophy caused by increased pressure load from hypertension, which often is associated with chronic renal disease. Left ventricular failure leads to pulmonary edema. Chronic alcoholism is most often associated with dilated cardiomyopathy. Diabetes mellitus accelerates atherosclerosis, leading to ischemic heart disease and myocardial infarction; the normal glucose level does not fit with diabetes mellitus. Hemochromatosis leads to dilated cardiomyopathy. Pneumoconioses produce restrictive lung disease with cor pulmonale and predominantly right ventricular hypertropy.

A 59-year-old man has experienced chronic fatigue for the past 18 months. On physical examination, he is afebrile. A chest radiograph shows bilateral pulmonary edema and a prominent left heart border. The representative gross appearance of his heart is shown in the figure. Laboratory studies show serum glucose, 74 mg/dL; total cholesterol, 189 mg/dL; total protein, 7.1g/dL; albumin, 5.2 g/dL; creatinine, 6.1 mg/dL; and urea nitrogen, 58 mg/dL. What is the most likely diagnosis? A. chronic alcoholism B. diabetes mellitus C. hemochromatosis D. pneumoconiosis E. systemic hypertension

E Histologic sections from malignant neoplasms are frequently assessed with a panel of immunostains to detect antigenic characteristics, such as protein expression, to aid in characterizing the cell of origin, as well as provide infor- mation in selection of treatment protocols. In this case, the immunostains are likely to reveal that this neoplasm is a high-grade urothelial carcinoma. A cytology smear shows light microscopic findings helpful to screen for malignancy, but the findings often fall short of diagnosing a specific cell type. The other listed techniques are best for determination of treatment and prognosis.

A 59-year-old man has noticed blood in his urine for the past week. Cystoscopy shows a 4-cm exophytic mass involv- ing the right bladder mucosa near the trigone. After biopsy specimens are obtained, he undergoes a radical cystectomy. Examination of the excised specimen shows an anaplastic car- cinoma that has infiltrated the bladder wall. Which of the fol- lowing techniques applied to the cells from his neoplasm is most likely to categorize the cell of origin? A Chromosomal karyotyping B Cytologic smear C DNA microarray D Flow cytometric analysis E Immunohistochemistry

The answer is D: Cytokeratins. Tumor markers are products of malignant neoplasms that can be detected in cells or body fluids. Useful tumor markers include immunoglobulins, fetal proteins, enzymes, hormones, and cytoskeletal proteins. Carcinomas uniformly express cytokeratins, which are intermediate filaments. Alpha-fetoprotein (choice A) is a marker for yolk sac carcinoma and hepatocellular carcinoma. Calretinin (choice B) provides a marker for mesothelioma. Carcinoembryonic antigen (choice C) is a marker for colon carcinoma and many other malignancies. Synaptophysin (choice E) is a marker for neuroendocrine tumors, including small cell carcinoma of the lung. Diagnosis: Squamous cell carcinoma of lung

A 60-year-old man presents with a 4-month history of increasing weight loss, wheezing, and shortness of breath. He has smoked two packs of cigarettes a day for 40 years. His past medical history is significant for emphysema and chronic bronchitis. A chest X-ray shows a 10-cm mass in the left lung. Bronchoscopy discloses obstruction of the left mainstem bronchus. A biopsy is obtained (shown in the image). Immunohistochemical studies of this biopsy specimen would most likely show strong expression of which of the following tumor markers? (A) Alpha-fetoprotein (B) Calretinin (C) Carcinoembryonic antigen (D) Cytokeratins (E) Synaptophysin

B A large, irregular, ulcerated mass such as that described is most likely malignant, and the epithelium of the bladder gives rise to carcinomas. Urothelial carcinomas are associated with smoking. An adenoma is a benign epithelial neoplasm of glandular tissues. A fibroma is a benign mesenchymal neoplasm. A papilloma is a benign, localized mass that has an exophytic growth pattern. A sarcoma is derived from cells of mesenchymal origin; sarcomas are much less common than carcinomas.

A 62-year-old man has had several episodes of hematuria in the past week. He has a 48 pack-year history of smoking cigarettes. On physical examination, there are no abnormal findings. A urinalysis shows 4+ hematuria, and cytologic ex- amination of the urine shows that atypical cells are present. A cystoscopy is performed and a 4-cm sessile mass with a nodular, ulcerated surface is seen in the dome of the bladder. Which of the following terms best describes this lesion? A Adenoma B Carcinoma C Fibroma D Papilloma E Sarcoma

E Brain often associated with liquefactive necrosis. Abscess - indicates liquefactive

A 64-year-old woman presents with fever, chills, headache, neck stiffness, vomiting, and confusion. The Kernig sign (passive knee extension eliciting neck pain) and Brudzinski sign (passive neck flexion eliciting bilateral hip flexion) are both positive. Examination of the cerebrospinal fluid reveals changes consistent with bacterial meningitis, and brain imaging demonstrates a localized abscess. Which of the following types of necrosis is most characteristic of abscess formation? (A) Caseous (B) Coagulative (C) Enzymatic (D) Fibrinoid (E) Liquefactive

E Pharmacogenomic testing may reveal polymorphisms that affect drug metabolism. The cytochrome P-450 system in hepatocytes is involved in metabolism of many substances, including drugs. Detection of polymorphisms can provide information about modification of drug dosing, or the need for a different drug. Trinucleotide repeat mutations may af- fect gene expression, such as the huntingtin gene, but unlike- ly those involved in drug metabolism. Changes in DNA lead to alterations in mRNA expression, but detecting the DNA alterations directly tends to be a more effective testing strat- egy. FISH analysis is useful in identifying altered regions of chromosomes, not single genes abnormalities. Autosomal re- cessive conditions may affect enzyme levels, such G6PD, but polymorphisms may not affect enzyme or substrate levels that can be reliably detected.

A 66-year-old man has been prescribed clopidogrel to help prevent future acute coronary events. He states that his father, aunt, and brother took this drug and had adverse side effects, including excessive bleeding, when prescribed the recommended dose. Which of the following genetic tests will help determine the most appropriate dose of clopidogrel for this man? A Enumeration of tandem repeat sequences B Expression profiling of mRNA C FISH analysis for the karyotype D Measurement of an enzyme level E Probing for a cyp450 polymorphism

E The high lipid content of central nervous system tissues results in liquefactive necrosis as a consequence of ischemic injury, as in this case of stroke. Apoptosis affects single cells and typically is not grossly visible. Coagulative necrosis is the typical result of ischemia in most solid or- gans. Fat necrosis is seen in breast and pancreatic tissues. Karyolysis refers to fading away of cell nuclei in dead cells.

A 68-year-old woman suddenly lost consciousness and on awakening 1 hour later, she could not speak or move her right arm. Two months later, a head CT scan showed a large cystic area in the left parietal lobe. Which of the following pathologic processes has most likely occurred in her brain? A Apoptosis B Coagulative necrosis C Fat necrosis D Karyolysis E Liquefactive necrosis

The answer is A: Abnormal pattern of cellular maturation. Cells that compose an epithelium exhibit uniformity of size and shape, and they undergo maturation in an orderly fashion (e.g., from plump basal cells to fl at superficial cells in a squamous epithelium). When we speak of dysplasia, we mean that this regular appearance is disturbed by (1) variations in the size and shape of the cells; (2) enlargement, irregularity, and hyperchromatism of the nuclei; and (3) disorderly arrangement of the cells within the epithelium. Dysplasia of the bronchial epithelium is a reaction of respiratory epithelium to carcinogens in tobacco smoke. It is potentially reversible if the patient stops smoking but is considered preneoplastic and may progress to carcinoma. Choices B, D, and E are not preneoplastic changes. Invasiveness (choice C) connotes malignant behavior.

A 68-year-old woman with a history of heavy smoking and repeated bouts of pneumonia presents with a 2-week history of fever and productive cough. A chest X-ray reveals a right lower lobe infi ltrate. A transbronchial biopsy confi rms pneumonia and further demonstrates preneoplastic changes within the bronchial mucosa. Which of the following best characterizes the morphology of this bronchial mucosal lesion? (A) Abnormal pattern of cellular maturation (B) Increased numbers of otherwise normal cells (C) Invasiveness through the basement membrane (D) Transformation of one differentiated cell type to another (E) Ulceration and necrosis of epithelial cells

A The M1 designation indicates that distant metasta- ses are present. N1 means local lymph nodes are positive for carcinoma. Elevated corticotropin levels indicate secretion of an ectopic hormone that may produce a paraneoplastic syndrome, but this is not part of staging. A T2 designation indicates that the overall size of the tumor is not large; it is still within the lung parenchyma and not impinging upon margins of the lung. The TNM system is used for staging, not microscopic grading of cellular differentiation.

A 69-year-old man has noted a chronic cough for the past 3 months. On physical examination, there is mild stri- dor on inspiration over the right lung. A chest radiograph shows a 5-cm right hilar lung mass, and a fine-needle aspi- ration biopsy specimen of the mass shows cells consistent with squamous cell carcinoma. If staging of this neoplasm is denoted as T2N1M1, which of the following findings is most likely in this man? A Brain metastases B Elevated corticotropin level C Infiltration of the chest wall D Obstruction of a mainstem bronchus E Poorly differentiated tumor cells

C Lung and hilar lymph nodes accumulate anthracotic pigmentation when carbon pigment is inhaled from polluted air. The tar in cigarette smoke is a major source of such car- bonaceous pigment. Older individuals generally have more anthracotic pigment, but this is not inevitable with aging— individuals living in rural areas with good environmental air quality have less pigment. Resolution of hemorrhage can produce hemosiderin pigmentation, which imparts a brown color to tissues. Hepatic failure may result in jaundice, char- acterized by a yellow color in tissues. Metastases are mass lesions that impart a tan-to-white appearance to tissues.

A 69-year-old woman has had a chronic cough for the past year. A chest radiograph shows a 6-cm mass in the left lung. A needle biopsy specimen of the mass shows carcinoma. A pneumonectomy is performed, and examination of the hilar lymph nodes reveals a uniform, dark black cut surface. Which of the following factors most likely accounts for the appear- ance of these lymph nodes? A Aging effects B Bleeding disorder C Cigarette smoking D Liver failure E Multiple metastases

D The RAS oncogene is the most common oncogene involved in the development of human cancers. Mutations of the RAS oncogene reduce GTPase activity, and RAS is trapped in an activated GTP-bound state. RAS then signals the nucleus through cytoplasmic kinases. The APC gene can cause activation of the WNT signaling pathway. The MYC oncogene is a transcriptional activator that is overex- pressed in many tumors. The p53 tumor suppressor gene encodes a protein involved in cell cycle control. The RET proto-oncogene encodes a receptor tyrosine kinase involved in neuroendocrine cells of the thyroid, adrenal medulla, and parathyroids. The sis oncogene encodes platelet-derived growth factor receptor-β, which is overexpressed in certain astrocytomas.

A 70-year-old woman reported a 4-month history of a 4-kg weight loss and increasing generalized icterus. On physical examination, she has midepigastric tenderness on palpation. An abdominal CT scan shows a 5-cm mass in the head of the pancreas. Fine-needle aspiration of the mass is performed. On biochemical analysis, the neoplastic cells show continued activation of cytoplasmic kinases. Which of the following genes is most likely to be involved in this process? A APC B MYC C p53 D RAS E RET F sis

C Autophagy is a form of cellular downsizing in re- sponse to stress, as the cell consumes itself, by upregulat- ing Atgs genes. Lipofuscin granules are residual bodies left over from this process. Cell death may eventually be triggered by autophagy, but by a different mechanism than apoptosis, a form of single cell necrosis in which cell frag- mentation occurs. Cancer cells acquire the ability to prevent autophagy, perhaps by downregulating PTEN gene expres- sion, and maintain a survival advantage even as the patient is dying. There is slow autophagy with aging, but autoph- agy is accelerated with stressors such as malnutrition and chronic disease. Hyaline is a generic term for intracellular or extracellular protein accumulations appearing pink and homogeneous with H&E staining. Karyorrhexis is nuclear fragmentation in a necrotic cell.

A 71-year-old man diagnosed with pancreatic cancer is noted to have decreasing body mass index. His normal con- nective tissues undergo atrophy by sequestering organelles and cytosol in a vacuole, which then fuses with a lysosome. However, the cancer continues to increase in size. Which of the following processes is most likely occurring in the normal cells but is inhibited in the cancer cells of this man? A Aging B Apoptosis C Autophagy D Hyaline change E Karyorrhexis

D Nodular prostatic hyperplasia (also known as benign prostatic hyperplasia [BPH]) is a common condition in older men that results from proliferation of both prostatic glands and stroma. The prostate becomes more sensitive to andro- genic stimulation with age. This is an example of pathologic hyperplasia. Apoptosis results in a loss of, not an increase in, cells. Dysplasia refers to disordered epithelial cell growth and maturation. Fatty change in hepatocytes may produce hepatomegaly. Although BPH is often called "benign pros- tatic hypertrophy," this term is technically incorrect; it is the number of glands and stromal cells that is increased, rather than the size of existing cells. A change in the glandular epi- thelium to squamous epithelium around a prostatic infarct would be an example of metaplasia.

A 71-year-old man has had difficulty with urination, in- cluding hesitancy and increased frequency, for the past 5 years. A digital rectal examination reveals that his prostate gland is palpably enlarged to twice normal size. A transurethral resection of the prostate is performed, and the microscopic appearance of the prostate "chips" obtained is that of nodules of glands with intervening stroma. Which of the following pathologic processes has most likely occurred in his prostate? A Apoptosis B Dysplasia C Fatty change D Hyperplasia E Hypertrophy F Metaplasia

B The valve is stenotic because of nodular deposits of calcium. The process is "dystrophic" because calcium deposition occurs in damaged tissues. The damage in this patient is a result of excessive wear and tear with aging. Amyloid deposition in the heart typically occurs within the myocardium and the vessels. Hereditary hemochromatosis is a genetic defect in iron absorption that results in extensive myocardial iron deposition (hemosiderosis). Hyaline change is a descriptive term used by histologists to describe protein deposits that are glassy and pale pink. The amount of lipofuscin increases within myocardial fibers (not valves) with aging.

A 72-year-old man died suddenly from congestive heart failure. At autopsy, his heart weighed 580 g (normal 330 g) and showed marked left ventricular hypertrophy and minimal coronary arterial atherosclerosis. A serum chemistry panel ordered before death showed no abnormalities. Which of the following pathologic processes best accounts for the appearance of the aortic valve seen in the figure? A Amyloidosis B Dystrophic calcification C Hemosiderosis D Hyaline change E Lipofuscin deposition

A. Normal cells handle physiologic demands and maintain metabolic functions within narrow ranges, termed homeostasis. Under disease conditions with stress on cells, there is adaptation to a new steady state. In this case, the loss of renal function leads to a higher urea nitrogen level as well as retention of fluid. The diuretic induces loss of the excess fluid to yield a new steady state. The protein restriction reduces urea nitrogen excretion, which also leads to a new steady state. Both are adaptations. Apoptosis refers to single cell necrosis in response to injury. An irreversible injury leads to cell death, but the changes described here are not evidence for cellular necrosis. The metabolism of cells is maintained for adaptation, with response to the diuretic and to protein restriction.

A 77 year old woman has chronic renal failure. Her serum urea nitrogen is 40 mg/dL. She is given a diuretic medication and loses 2 kg (4.4 lb). She reduces the protein in her diet and her serum urea nitrogen decreases to 30 mg/dL. Which of the following terms best describes cellular responses to disease and treatment in this woman? A. Adaptation B. Apoptosis C. Necroptosis D. Irreversible injury E. Metabolic derangement

B The infant has DiGeorge syndrome, resulting from a chromosome 22q11.2 microdeletion. This is indicated in the metaphase spread by the presence of only three dots be- cause this region is deleted on one chromosome 22, but both number 22 chromosomes are present. Hyperdiploidy is more than 46 chromosomes. With aneuploidy, there is an abnor- mal number of chromosomes (trisomy, monosomy), and loss or gain of autosomes tends to produce fetal loss except for some cases of trisomies 13, 18, and 21 and monosomy X. A chromosome inversion would shift the marked region to a different part of the same chromosome. In monosomy, only one of a pair of chromosomes is present. A translocation is the swapping of genetic material between two chromosomes.

A 9-month-old infant has had numerous viral and fungal infections since birth. On physical examination, no congenital anomalies are noted. Laboratory studies show hypocalcemia. FISH analysis of the infant's cells is performed. A metaphase spread is shown in the figure, with probes to two different regions on chromosome 22. Which of the following cytogenetic abnormalities is most likely to be present? A Hyperdiploidy B Deletion C Inversion D Monosomy E Translocation

F In the figure the disorderly, atypical epithelial cells involve the entire thickness of the epithelium. They extend through the underlying basement membrane and into the underlying stroma as rounded nests at the right, a process known as invasion. The ectocervix and the squamous metaplasia of endocervix give rise to dysplasia from which squamous cell carcinoma can arise. Carcinoma in situ is confined to the epithelium; if the basement membrane is breached, the lesion is no longer in situ, but rather invasive. An adenocarcinoma is a malignant neoplasm arising from glandular epithelium, such as the endocervix or endometrium, not the ectocervix. A dysplastic process could precede development of carcinoma in situ and squamous carcinoma; dysplasia involves only part of the thickness of the epithelium. A hamartoma contains a mixture of cell types common to a tissue site. Metaplasia can occur in response to persistent infection with human papillomavirus (HPV) and other inflammatory conditions. Metaplasia can be the precursor to dysplasia.

A Pap smear obtained from a 29-year-old woman during a routine health maintenance examination is abnormal. She is currently asymptomatic. She has a history of multiple sexual partners. Cervical biopsy specimens are obtained and the microscopic appearance is shown in the figure. Which of the following is the most likely diagnosis? A Adenocarcinoma B Carcinoma in situ C Dysplasia D Hamartoma E Metaplastic transformation F Squamous cell carcinoma

D The human genome can be divided into blocks of DNA with varying numbers of contiguous single nucleotide polymorphisms (SNPs) that form haplotypes and can clus- ter from linkage disequilibrium, so that similar haplotypes suggest shared inheritance. The use of chips with more than a million SNPs can identify small variations in DNA from person to person, and linkage of these polymorphisms to disease can help narrow the search for candidate genes whose altered function may relate to a disease. Fluorescence in situ hybridization (FISH) probes aid in identifying specific chromosome regions and can identify abnormalities such as chromosomal deletions and translocations. The standard karyotype with banding provides information about chro- mosome number and major structural alterations, but does not identify specific genes or their loci. Analysis of mRNA expression provides a roundabout way of determining de- rivative DNA alterations, but DNA is easier to work with directly. Southern blotting has been largely supplanted by other techniques, but is useful for detection of trinucleotide repeat expansions and clonal gene rearrangements.

A case-control study is performed involving persons di- agnosed with essential hypertension. Genetic analysis reveals linkage disequilibrium. Haplotypes of affected persons differ from the controls in the chromosome containing the angioten- sinogen gene. Which of the following types of genetic analysis is most likely to yield this information? A Fluorescence in situ hybridization B Giemsa banded karyotyping C RNA expression D Single nucleotide polymorphisms E Southern blotting

C Familial hypercholesterolemia results from mutations in the LDL receptor gene, causing plasma LDL cholesterol to increase because it is not catabolized or taken up by the liver. It is an autosomal dominant disorder with a carrier rate of 1 in 500, so the frequency of homozygosity is 1 in 1 million. Heterozygotes have total serum cholesterol levels twice nor- mal; homozygotes have levels even higher, with death from myocardial infarction by the second decade. The statin drugs inhibit the HMG-CoA reductase and reduce cholesterol levels in heterozygotes. Steroid hormone receptors, such as those for cortisol, are located in the cell nucleus. Insulin receptors play a role in glucose metabolism and glycemic control that may be part of diabetes mellitus with risk for atherosclerosis; statin drugs have no effect on diabetes mellitus. Abnormal leptin receptors may play a role in some forms of obesity. TGF-α is a growth factor with a role in inflammation, cell proliferation, and repair.

A clinical study is undertaken with subjects from fami- lies in which complications of atherosclerotic cardiovascular disease and tendinous xanthomas occurred before age 30 years. Some of the children in these families are observed to have early atheroma formation. These affected individuals benefit from treatment with pharmacologic agents that inhibit HMG-CoA reductase. Affected individuals in these families are most likely to have a mutation in a gene encoding a cell surface receptor for which of the following? A Cortisol B Insulin C LDL cholesterol D Leptin E TGF-α

E 2/3 x ½ x 2/3 x ½ = 1/9

A couple of European decent come to your office for counseling. They wish to start a family, but both prospective parents have siblings that are affected by cystic fibrosis. Assuming that the carrier frequency for cystic fibrosis is 1/30 in the European decent, what is the likelihood that their first child will be affected by cystic fibrosis? A. 1/3600 B. 1/1800 C. 1/900 D. 1/90 E. 1/9

D Most inborn errors of metabolism involve mutations in genes encoding for enzymes. Because one active allele produces half the needed enzyme, this is likely sufficient to avoid disease. Inheritance of two mutant alleles, one from each parent, is required for appearance of disease, so the pattern is autosomal recessive, and the recurrence risk is 25%. Most autosomal recessive genes are infrequent in the population, so a family history is unlikely. Even if 1 in 10 persons carries the mutant recessive gene, a homozygote will be 1 in 400. The standard recurrence risk for any pregnancy is 3%. The recurrence risk is increased to 7% in diseases such as diabetes mellitus, or when a syndrome is identified without a defined inheritance pattern, or with multifactorial inheritance. Autosomal dominant conditions usually result from mutations in genes encoding for structural genes and have a recurrence risk of 50%.

A female infant born at term shows failure to thrive and failure to achieve developmental milestones. A pedigree reveals only this child is affected out of four generations on both sides of the family. Tissue fibroblasts obtained from this child shows a 46,XX karyotype. Cultured fibroblasts show accumulation of an intermediate product in a metabolic pathway in which multiple enzymes are involved. What is the most likely recurrence risk for this condition in siblings of this infant? A 3% B 8% C 15% D 25% E 50% F 100%

E Almost all of the normal genetic material is present in the case of a Robertsonian translocation because only a small amount of the p arm from each translocated chromosome is lost. The maternal karyotype is 45,XX,t(14;21). Statistically, one of six fetuses in a mother who carries a Robertsonian translocation will also be a carrier. In balanced reciprocal translocation, the same possibility of inheriting the defect exists. The other listed structural abnormalities are likely to result in loss of significant genetic material, reducing surviv- ability, or to interfere with meiosis.

A healthy 20-year-old woman, G3, P2, Ab1, has previ- ously given birth to a liveborn infant and a stillborn infant, both with the same karyotypic abnormality. On physical examina- tion, she is at the 50th percentile for height and weight. She has no physical abnormalities noted. Which of the following karyo- typic abnormalities is most likely to be present in this woman? A Deletion of q arm - del(22q) B Isochromosome - 46,X,i C Paracentric inversion - inv(18) D Ring chromosome - r(13) E Robertsonian translocation - t(14;21)

B

A low magnification view of the esophagus is shown. Region 1 is shown at higher magnification on the right, and region 2 is shown at higher magnification below. Region 1 highlights the mucosa. The epithelium is A. simple columnar. B. stratified squamous, non-keratinized. C. pseudostratified. D. stratified squamous, keratinized. E. transitional.

The answer is E: Hypomere. Skeletal muscles are derived from the paraxial mesoderm. Most of the paraxial mesoderm segregates into somites, which in turn give rise to myotome, dermatome, and sclerotome units. The myotome is the muscle-producing component. Each myotome splits into two segments: a more posterior epimere and a more anterior hypomere. The hypomere gives rise to the muscles in the anterolateral body wall (including the abdominal wall) and the limbs, whereas the epimere forms the intrinsic muscles of the back. The hypomere (and its derived muscles) is inner- vated by anterior (ventral) primary rami of spinal nerves. The epimere (and its derivatives) is supplied by posterior (dorsal) primary rami of spinal nerves. Choice A (Surface ectoderm) is incorrect. The ectoderm does not produce skeletal muscles. However, it does form a few small smooth muscles, for exam- ple, the dilator and sphincter muscles in the pupil of the eye. In general, the ectoderm forms the outer surface of the body and structures that form as invaginations from the surface, for example, the epidermis (including the hair and nails), subcutaneous glands (including the mammary glands), and the neural tube. Choice B (Intermediate mesoderm) is incorrect. Most of the urinary and genital systems develop from the intermediate mesoderm. This small zone connects the paraxial and lateral plate mesodermal areas. It gives rise to the excretory parts of the urinary system, the gonads, and much of the genital duct work. Choice C (Splanchnic mesoderm) is incorrect. The splanchnic mesoderm is the visceral layer of the lateral plate mesoderm. It becomes closely invested with the endoderm and gives rise to cardiac muscle that forms the myocardium and to the smooth muscle in the wall of the gut tube and its derivatives. Choice D (Epimere) is incorrect. The epimere is the posterior component of the myotome, which forms the deep (intrinsic) layer of back musculature. These muscles are supplied by posterior (dorsal) primary rami of spinal nerves. The abdominal wall musculature, affected in prune belly syndrome, develops from the hypomere, not the epimere.

A male infant presents with prune belly syndrome, as shown in the given photo. In this syndrome, the abdominal wall musculature is poorly developed, resulting in an abdominal wall so thin that the internal organs are visible and easily palpable. The muscles affected in this condition develop from which of the following embryonic sites? (A) Surface ectoderm (B) Intermediate mesoderm (C) Splanchnic mesoderm (D) Epimere (E) Hypomere

B. A hemangioma is a vascular tumor that can be present at birth in which the abnormal proliferation of blood vessels leads to a mass resembling a neoplasm. Hemangiomas are mesodermal in origin, in that they are formed by embryonic blood cells and the vascular endothelium formed by angioblasts.

A male newborn has a hemangioma on the left frontotemporal region of his face and scalp. The cells forming the hemangioma are derived from which of the following cell layers? (A) Ectoderm only (B) Mesoderm only (C) Endoderm only (D) Ectoderm and mesoderm (E) Endoderm and mesoderm

E. Jewish descent have a high carrier frequency for BRCA1 and 2 mutation alleles. Reduced penetrance indicates that the grandmother can be a carrier even though she did not get breast cancer. Same is true for his mother. Since there is no knowledge of transmission, the risk of transmission is just ½ per generation.

A male patient, John, enters your clinic. A family history reveals that he is of Jewish descent and his maternal great grandmother, but not grandmother, died from breast cancer. John's mother (62 year old) is still alive and has not suffered from breast or ovarian cancer. John has is excited that he is soon to be a father with a baby girl, Jane. How do you counsel John with regard to the Jane's risk of getting breast cancer? (Note, the BRCA1 and 2 mutations are inherited in an dominant manner with reduced penetrance.) Choose the best answer. A. There is little to no risk for Jane. B. Assuming that John's great grandmother had a defect in the BRCA1 or BRCA2 gene, Jane has at most a 1/2 chance of inheriting the disease gene. C. Assuming that John's great grandmother had a defect in the BRCA1 or BRCA2 gene, Jane has at most a 1/4 chance of inheriting the disease gene. D. Assuming that John's great grandmother had a defect in the BRCA1 or BRCA2 gene, Jane has at most a 1/8 chance of inheriting the disease gene. E. Assuming that John's great grandmother had a defect in the BRCA1 or BRCA2 gene, Jane has at most a 1/16 chance of inheriting the disease gene

E. 1 x 1 x 1 x 1/2 = 1/2

A mother has sickle cell anemia while her husband is not affected. Both parents are of African American descent. They have 2 children, both of which have sickle cell anemia. What is the probability that their third child will be affected? Assume that that population carrier risk is African Americans is 1/10. A. 1/20 B. 1/10 C. 1/5 D. ¼ E. ½

E Has 2 different allelic mutations in the gene encoding hexosaminidase-A and the father is a carrier of a severe allelic form.)

A mother of Ashkenazi Jewish decent is afflicted with an adult onset from of TaySachs. Her husband, also of Ashkenazi Jewish decent, is unaffected. The carrier frequency of Tay_Sachs in the Ashkenazi Jewish population is 1/300. They have a child that has the Infantile form of Tay-Sachs. This is best explained by: A. This is due to anticipation. B. This is due to reduced penetrance. C. This is due to haploinsufficiency. D. The mother was a heterozygous for the wild type gene. E. The mother was a compound heterozygote.

B

A new signaling molecule has been found by a group of researchers in the Windy City Medical Institute. This molecule appears to activate a 7-transmembrane-domain cell surface receptor on endothelial cells. The binding of the factor results in the decrease of cyclic AMP in the cell cytosol. By what mechanism does this factor exert its effect on the endothelial cells? A. Activation of Gαs B. Activation of Gαi C. Activation of Gαq D. Activation of cAMP phosphodiesterase E. Activation of GTPase

C Calcium release is signaled by the Gαq-phosphoinositide cascade.

A new signaling molecule has been found by a group of researchers in the Windy City Medical Institute. This molecule appears to activate a 7-transmembrane-domain cell surface receptor on endothelial cells. The binding of the factor results in the release of calcium in the cell cytosol. By what mechanism does this factor exert its effect on the endothelial cells? A. Activation of Gαs B. Activation of Gαi C. Activation of Gαq D. Activation of tyrosine kinase E. Activation of STAT (Signal transducer and Activator of transcription)

B Aka trisomy 21, down syndrom

A newborn child shows a flattened face especially at the bridge of the nose, a shorter neck, ears, hands and feet, than normal. A karyotype is performed. Which karyotype represents this patient? A. 45, -15 +der[15q;21q] B. 46, -15 +der[15q;21q] C. 45, -21 +der[15q;21q] D. 46, -21 +der[15q;21q] E. 45, -15, -21, +der[15q;21q]

D

A not so young anymore professor has to give her very first lecture to medical students the next day. Being so nervous, she decides to take 5 mg of the anti-anxiety drug diazepam (weak base, pKa 3.3, half life ~200 h). Unfortunately, she left her diazepam at school, so she cannot take it until she arrives in the morning, about 30 minutes before her lecture starts. To speed-up the onset of action, and make sure that diazepam has already been absorbed by the time the lecture starts, this not so young anymore professor should: A. take diazepam on an empty stomach B. take diazepam on an full stomach C. take diazepam with lots of water D. take diazepam on an empty stomach and with lots of water E. take diazepam with orange juice, to make her urine more acidic F. take diazepam with sodium bicarbonate, to make her urine more basic

D Will be more ionized in milk, which is more basic.

A nursing mother has been taking an antibiotic orally (weak acid, pKa=5.4). Assuming that the pH of the blood is 7.4 and that of the milk is 8.4, what is the ratio of the ionized form of the antibiotic in the blood vs. the milk? A. 1000:1 B. 100:1 C. 10:1 D. 1:10 E. 1:100 F. 1:1000

E

A patient who just had his appendix removed is prescribed hydrocodone for post-operative pain management. After 5 days of daily treatment with hydrocodone (2-3 times/day, according to the pain level) hydrocodone-induced pain relief is substantially diminished. This is most likely due to: A. counterfeit incorporation B. heterologous desensitization C. rapid tachyphylaxis D. receptor supersensitivity and rebound effects E. tolerance and receptor down-regulation

D Whole exome sequencing limits the search for the roughly 2% of the genome that consists of protein-encoding exons responsible for as many as 80% of mendelian diseases, and the cost is significantly reduced compared to whole ge- nome sequencing. Pyrosequencing is a more sensitive varia- tion of Sanger sequencing that is most often used when testing for particular sequence variants in a background of normal alleles, such as tumor cells admixed with large numbers of stromal cells. Sanger sequencing is used together with PCR to allow sequencing of any known segment of DNA. Targeted sequencing is useful for identification of a single gene or panel of genes, either by subselecting relevant clones from a whole genome library via custom complementary probes, or by alter- nate preparations from genomic DNA such as multiplex PCR.

A pedigree reveals that multiple family members over four generations have been affected by the onset of conges- tive heart failure within the first four decades of life. A cardio- myopathy is suspected, but specific features of the disease are not known, and no prior genetic testing has been performed. Which of the following techniques involving DNA sequencing is most likely to identify a specific mutation in a cost-effective manner? A Pyrosequencing B Sanger sequencing C Targeted sequencing D Whole exome sequencing E Whole genome sequencing

A. Correct answer. Although there is not a net increase of sodium uptake by cells due to the sodium pump, radiolabeled sodium entering a cell may exchange with an unlabeled sodium molecule upon being pumped out, thus resulting in a modest loss of radiolabeled sodium from the extracellular fluid. Thus, the concentration of radiolabeled sodium in the plasma following equilibration will be less than expected, and this will calculate into a larger extracellular fluid volume than expected. B. Incorrect answer. See A. C. Incorrect answer. See A. Radiolabeled sodium measures extracellular fluid volume, but since some of it enters cells and exchanges with unlabeled sodium, it over-estimates extracellular fluid volume when measured in plasma. D. Incorrect answer. See A. Radiolabeled sodium distributes throughout both the interstitial and plasma volumes, and thus is a measurement of total extracellular fluid volume.

A physician wants to measure the volume of extracellular fluid in a patient with peripheral edema. Radiolabeled sodium is infused intravenously, and following equilibration, its concentration is measured in the plasma. Assuming that the labeled sodium is not lost from the body during the test, which of the following results would you expect? A. Extracellular fluid volume would be over-estimated. B. Extracellular fluid volume would be under-estimated. C. Only intracellular fluid volume would be measured, since sodium cannot enter cells. D. Since radiolabeled sodium is measured in the plasma, only plasma volume is calculated.

C

A pregnant mother comes into your clinic and has a history of repeated miscarriages. After consultation with the genetics lab, it is decided to perform prenatal testing to determine if there are any chromosomal abnormalities. The fetus shows a karyotype, 46, XX, -1, + der(1) t(1;22)(q25;q13). You will counsel your patient that the fetus has a: A. balance translocation and will be normal. B. partial trisomy in chromosome 1 and partial monosomy in chromosome 22. C. partial monosomy in chromosome 1 and partial trisomy in chromosome 22. D. partial trisomy in chromosome 1 and partial trisomy in chromosome 22. E. partial monosomy in chromosome 1 and partial monosomy in chromosome 22.

B

A pregnant mother comes into your clinic and has a history of repeated miscarriages. After consultation with the genetics lab, it is decided to perform prenatal testing to determine if there are any chromosomal abnormalities. The fetus shows a karyotype, 46, XX, -22, + der(22) t(1;22)(q25;q13). You will counsel your patient that the fetus has a: A. balance translocation and will be normal. B. partial trisomy in chromosome 1 and partial monosomy in chromosome 22. C. partial monosomy in chromosome 1 and partial trisomy in chromosome 22. D. partial trisomy in chromosome 1 and partial trisomy in chromosome 22. E. partial monosomy in chromosome 1 and partial monosomy in chromosome 22.

C Failure to find amplified product by PCR analysis in such a case could mean that the fetus is not affected, or that there is a full mutation that is too large to be detected by PCR. The next logical step is a Southern blot analysis of genomic DNA from fetal cells. Routine karyotyping of the amniotic fluid cells is much less sensitive than a Southern blot analysis. Karyotyping of the unaffected father cannot provide information about the status of the FMR1 gene in the fetus because amplification of the trinucleotide occurs during oogenesis. For the same reason, PCR analysis of the mother's FMR1 gene is of no value.

A pregnant woman with a family history of fragile X syndrome undergoes prenatal testing of her fetus. PCR analysis to amplify the appropriate region of the FMR1 gene is attempted using DNA from amniotic fluid cells, but no amplified products are obtained. Which of the following is the most appropriate next step? A Routine karyotyping of the amniotic fluid cells B Routine karyotyping of the unaffected father C Southern blot analysis of DNA from the amniotic fluid cells D PCR analysis of the mother's FMR1 gene E No further testing

A. There are forms of noncoding RNA that play a role in gene expression. Long noncoding RNA (lncRNA) segments greater than 200 nucleotides in length can bind to chromatin to restrict access of RNA polymerase to coding segments. The X chromosome transcribes XIST, a lncRNA that binds to and repress X chromosome expression. However, not all genes on the "inactive" X chromosome are switched off. The RNA transcribed from nuclear DNA that directs protein synthesis through translation is mRNA. MicroRNAs (miRNAs) are noncoding RNA sequences that inhibit the translation of mRNAs. Gene-silencing RNAs (small interfering RNAs siRNAs) have the same function as miRNAs, but they are produced synthetically for experimental purposes. Transfer RNA (tRNA) participates in the translation of mRNA to proteins by linking to specific amino acids.

A study of peripheral blood smears shows that neutrophil nuclei of women have a Barr body, whereas those of men do not. The Barr body is an inactivated X chromosome. Which of the following forms of RNA is most likely to play a role in Barr body formation? A. lncRNA B. mRNA C. miRNA D. siRNA E. tRNA

A Irreversible cellular injury is likely to occur when cytoplasmic calcium increases. Calcium can enter cells and also accumulate in mitochondria and endoplasmic reticulum. The excess calcium activates ATPases, phospholipases, proteases, and endonucleases, which injure cell components. Mitochondrial permeability is increased to release cytochrome c, which activates caspases leading to apoptosis. Of the other ions listed, sodium enters the cell, while potassium diffuses out when the sodium pump fails as ATP levels fall; but this is potentially reversible.

A tissue preparation is experimentally subjected to a hypoxic environment. The cells in this tissue begin to swell, and chromatin begins to clump in cell nuclei. ATPases are activated, and ATP production decreases. Which of the following ions accumulating in mitochondria and the cytosol contributes most to these findings and to eventual cell death? A Ca2+ B Cl− C HCO3− D K+ E Na+ F PO4 3−

D

About what percentage of Na channel inactivation gates are normally open in a nerve axon at rest? A. 5% B. 25% C. 50%. D. 75% E. 99%

Irreversible cell injury is associated with loss of mem- brane integrity. This allows intracellular enzymes such as AST and ALT to leak into the serum. All other morphologic changes listed are associated with reversible cell injury, in which the cell membrane remains intact and the cells do not die.

An 11-year-old girl becomes infected with hepatitis A and experiences mild nausea for 1 week. On physical exami- nation, she has minimal right upper quadrant tenderness and scleral icterus. Laboratory findings include a serum AST of 68 U/L, ALT of 75 U/L, and total bilirubin of 5.1 mg/dL. Her laboratory findings most likely result from which of the fol- lowing changes in her hepatocytes? A Cell membrane defects B Lysosomal autophagy C Mitochondrial swelling D Nuclear chromatin clumping E Ribosomal dispersion

E Plasma sodium is normal, so it's not 21-hydroxylase. X-linked Androgen Insensitivity Syndrome: develop testes but cannot react to androgen, externally female but infertile, blind vagina

An 18 year old girl complains that she has not had her first menstrual period. Palpation in the groin area reveals a growth, which is later identified as a testicle. Bulbar smear indicates a karyotype of 46, XY. Plasma sodium, potassium, and glucose levels are normal as are urinary levels of urinary 7-ketosteroid and 17- hydroxycorticosteroid. A deficiency in which of the following is most consistent with this disorder of sexual development? A. 21-hydroxylase (CYP21) B. 11-β-hydroxylase (CYP11B1) C. 17-α-hydroxylase (CYP17A1) D. 18-hydroxylase (CYP11B2) E. the androgen receptor

B An aberrant expression of receptor tyrosine kinase can cause breast cancer. DNA viral infection can be seen in HPV which is a DNA virus that can cause cervical cancer. Retroviral infections can manifest in T-cell leukemias/lymphomas.

An 8-year-old boy is referred to the dermatologist for numerous "suspicious" pigmented lesions on the face and neck. Further history reveals that the patient has had difficulty seeing out of his right eye; he is referred to the ophthalmologist, who diagnoses an ocular melanoma. Based on the patient's symptoms, the diagnosis of xeroderma pigmentosum is considered. This condition results from (A) aberrant expression of a receptor tyrosine kinase. (B) an inborn defect in DNA repair. (C) chemical carcinogenesis. (D) DNA viral infection. (E) retroviral infection

C

An anion (A- ) is being transported into an epithelial cell by a Na+ coupled-cotransporter with a stoichiometry of 1A per 1 Na+ . The cell has a membrane potential of -60 mV. Once a steady-state has been established what will happen to the intracellular concentration of the anion A if the membrane potential of the cell is clamped to -80 mV? A. will increase B. will decrease C. will not change

B A screening program should reliably detect early cancers with higher incidence. Breast cancer affects up to 1 in 9 women in these regions. Mammography may aid in detection of small cancers that have a better prognosis. A chest radiograph is an insensitive technique for detecting early lung cancers. Because Pap smear screening can detect dysplasias and in situ carcinomas that can be treated before progression to invasive lesions, deaths from cervical carcinoma have steadily decreased since this screening method became widely available in the last half of the 20th century. The introduction of human papillomavirus (HPV) vaccination will diminish the numbers of cervical cancers even further. Serum tumor markers have not proved useful as general screening techniques, although they are useful in selected circumstances. Use of stool guaiac has had a minimal effect on rates of death from colorectal carcinomas, but physicians are cautioned not to indicate "rectal deferred" on the physical examination report, and hence contribute to the problem. Urine cytology is better than urinalysis for detection of urothelial malignancies, but it does not have a high sensitivity.

An epidemiologic study analyzes health care benefits of cancer screening techniques applied to persons more than 50 years of age. Which of the following diagnostic screening techniques used in health care is most likely to have the great- est impact on reduction in cancer deaths in Europe and North America? A Chest radiograph B Mammography C Pap smear D Serum tumor markers E Stool guaiac F Urine cytology

E Development of colonic adenocarcinoma typically takes years, during which time multiple mutations occur within the mucosa, including mutations involving such genes as APC (adenomatous polyposis coli), K-RAS, and p53. The accumulation of mutations, rather than their occurrence in a specific order, is most important in the development of a carcinoma. Activation of proto-oncogenes, extensive re- generation, faulty TP53 genes, and amplification of growth factor receptor genes all contribute to the development of malignancies, but they are not sufficient by themselves alone to produce a carcinoma from an adenoma of the colon. Inher- ited loss of wild-type TP53 contributes to multiple cancers, but not to sporadic adenomas.

An epidemiologic study investigates the potential mor- phologic and molecular alterations that may contribute to the development of cancers in a population. Data analyzed from resected colonic lesions show changes that suggest the evo- lution of a sporadic colonic adenoma into an invasive carci- noma. Which of the following best describes the mechanism producing these changes leading to colonic malignancies? A Activation of proto-oncogenes by chromosomal translocation B Extensive regeneration of tissues increasing the mutation rate in regenerating cells C Inheritance of defects in TP53 genes that increase the susceptibility to develop cancer D Overexpression of growth factor receptor genes E Stepwise accumulation of multiple proto-oncogene and tumor suppressor gene mutations

CDK1 controls an extremely important transition point, the G2 to M transition, during the cell cycle, which can be regulated by CDK inhibitors. The other checkpoints listed are regulated by a distinct set of proteins.

An experiment analyzes factors involved in the cell cycle during growth factor-induced cellular regeneration in a tissue culture. Cyclin B synthesis is induced; the cyclin B binds and activates cyclin-dependent kinase 1 (CDK1). The active kinase produced by this process is most likely to control progression in which of the following phases of the cell cycle? A. G0 to G1 B. G1 to S C. S to G2 D. G2 to M E. M to G1

B These histologic findings are typical of apoptosis. The BCL2 gene product inhibits cellular apoptosis by bind- ing to Apaf-1. Hence, the knockout removes this inhibition The BAX gene product promotes apoptosis, and a knockout would protect against apoptosis. The C-MYC gene is in- volved with oncogenesis. The FAS gene encodes for a cellular receptor for Fas ligand that signals apoptosis. Activity of the p53 (TP53) gene normally stimulates apoptosis, but mutation favors cell survival.

An experiment introduces a knockout gene mutation into a cell line. The frequency of shrunken cells with chromatin clumping, karyorrhexis, and cytoplasmic blebbing is increased compared with a cell line without the mutation. Overall sur- vival of the mutant cell line is reduced. Which of the following genes is most likely to be affected by this mutation? A BAX B BCL2 C C-MYC D FAS E p53

B An increase in flow rate at constant pressure indicates that the artery has dilated and its internal diameter has increased. Therefore, you need to look for an agonist that causes dilation of the artery and the second messenger that it generates. The only correct answer is EPI acting on β2 receptor to increase adenylyl cyclase activity and elevate cAMP. In smooth muscle, cAMP causes relaxation by PKA-mediated phosphorylation of MLCK.

An isolated arterial preparation is perfused with a physiological solution at a constant pressure. A neurotransmitter (receptor agonist), when added to the perfusion solution, caused the flow rate to increase. Which one of the following agonist-receptor signaling pathways is most likely to be predominant in this vessel? A. Norepinephrine - α1 receptor - phospholipase C B. Epinephrine - β2 receptor - adenylyl cyclase C. Epinephrine - α1 receptor - adenylyl cyclase D. Epinephrine - β2 receptor - phospholipase C E. Acetylcholine - α1 receptor - adenylyl cyclase

C Muscarinic receptor antagonist should block vasodilation produced by parasympathetic stimulation in some tissues. α1 agonist should cause vasoconstriction by elevating [Ca2+] in the cell. NO stimulates guanylyl cyclase to form cGMP to cause vasodilation. β1 agonist elevates cAMP in smooth muscle to cause relaxation. Norepinephrine has both α / β1 effects. In blood vessels, mainly a1 receptor present, resulting in vasoconstriction (IP3 production).

An unknown substance was found to produce vasodilation and elevation of cGMP in the male genitalia. What is the most likely identity of the substance? A. Atropine (muscarinic receptor blocker) B. Phenylephrine (α1 agonist) C. Nitroprusside (NO generating compound) D. Albuterol (β 2 agonist) E. Norepinephrine (α / β 1 receptor agonist)

D Lipofuscin is a "wear-and-tear" pigment that in- creases with aging, particularly in liver and myocardium. This granular golden brown pigment seen adjacent to the myocyte nucleus in the figure has minimal effect on cellular function in most cases. Rarely, there is marked lipofuscin deposition in a small heart, a so-called brown atrophy. Bilirubin, another breakdown product of hemoglobin, imparts a yellow appearance (icterus) to tissues. Hemosiderin is the breakdown product of hemoglobin that contains the iron. Hearts with excessive iron deposition tend to be large. Glycogen is increased in some inherited enzyme disorders, and when the heart is involved, heart size increases. Melanin pigment is responsible for skin tone: the more melanin, the darker the skin.

At autopsy, the heart of a 63-year-old man weighs only 250 g (normal 330 g) and has small right and left ventricles. The myocardium is firm, with a dark chocolate-brown color throughout. The coronary arteries show minimal atherosclerotic changes. An excessive amount of which of the following substances, shown in the figure, would most likely be found in the myocardial fibers of this heart? A Bilirubin B Glycogen C Hemosiderin D Lipofuscin E Melanin

D GDP-GTP exchange is needed in G-proteins, which is not associated with the ANFR signaling

Atrial natriuretic factor (ANF) is a cardiovascular hormone mainly secreted by the cells in the heart atrium in response to stretching forces by the increased blood volume. ANF binds to a cell surface receptor called the atrial natriuretic factor receptor (ANFR), which is expressed in various tissues including the kidney, adrenal and heart. ANFR has an intracellular guanylate cyclase domain. Which one of the following is NOT required for transduction of the signal by ANFR? A. Hormone binding to the receptor. B. Conformational change in the receptor following hormone binding. C. Dimerization of the receptor. D. GDP-GTP exchange on the intracellular domain. E. Binding of GTP to the cytosolic domain of the receptor.

C ¼ x ½ = 1/8

Ben has married Marge (arrow) and they are planning on having children. Marge has a family history of Becker's muscular dystrophy and they are concerned that their children will inherit the disease. What is the probability that Ben and Marge's first boy will be born with muscular dystrophy? (from above) A. 1/32 B. 1/16 C. 1/8 D. ¼ E. ½

B ¼ x ½ x ½ = 1/16

Ben has married Marge (arrow) and they are planning on having children. Marge has a family history of Becker's muscular dystrophy and they are concerned that their children will inherit the disease. What is the probability that Ben's and Marge's first child will be born with muscular dystrophy? (from above) A. 1/32 B. 1/16 C. 1/8 D. ¼ E. ½

C

Ben has married Marge (arrow) and they are planning on having children. Marge has a family history of Becker's muscular dystrophy and they are concerned that their children will inherit the disease. How many people in this pedigree are obligate carriers? A. 4 B. 5 C. 6 D. 7 E. 8

D

Burkitt's lymphoma (B-cell malignancy) and chronic myelogenous leukemia (CML) are blood cancers. Select the one statement that best describes these cancers. A. Each cancer involves inappropriate activation of a protein tyrosine kinase. B. Each cancer involves inappropriate activation of a transcription factor. C. Each cancer involves point mutations of an oncogene. D. Proto-oncogenes are specifically activated at the break point in reciprocal chromosomal translocations in each of these disorders. E. A phosphatase inhibitor has been developed as an effective therapeutic treatment for Burkitt's lymphoma.

D

Chad is a two-year-old boy who has come for well child care. At Chad's last well child visit, at 15 months of age, the history and physical examination were normal, but he was not talking. Chad still has no expressive language. He appears to hear normally and follows simple commands. His gross and fine motor development appear normal for his age. He is described as shy. Parents note that he is not communicative with other children or adults and consistently avoids eye contact. He is frequently irritable and hyperactive at home. The parents wonder if he is autistic. What is the most likely diagnosis for Chad? A. Rett syndrome with reduced penetrance B. Down syndrome with reduced penetrance C. Smith-Magenis syndrome D. Fragile X E. William-Beuren syndrome

A. All primary oocytes are formed by month 5 of fetal life, so no oogonia are present at birth.

Concerning maturation of the female gamete (oogenesis), when do the oogonia enter meiosis I and undergo DNA replication to form primary oocytes? (A) During fetal life (B) At birth (C) At puberty (D) With each ovarian cycle (E) Following fertilization

1/144 1/6*1/2*1/6*1/2=1/144

Dennis and Dawn are second cousins. They would like to get married, but they are concerned that they might have a child with Congenital adrenal hyperplasia. (Loss of function, not haplo-insufficient) You do a family history and get a pedigree shown below. What is the likelihood that there first child will get Congenital adrenal hyperplasia?

A An agonist can mimic the ligand (e.g., a hormone). Thus, it can bind to the corresponding hormone receptor. Dexamethasone can target the cells in the hypothalamus or pituitary to reduce the secretion of CRH (corticotropin-releasing hormone; CRH activates the secretion of ACTH in the pituitary by binding to a GPCR) and ACTH (adrenocorticotropic hormone; ACTH activates the production of cortisol in adrenal cortex by binding to a GPCR), respectively. The receptors in the hypothalamus and the anterior pituitary (for dexamethasone or cortisol) that mediate the negative feedback inhibition appear to be certain cell surface receptors (likely to be GPCR).

Dexamethasone is a cortisol agonist that normally suppresses the production of cortisol by feedback inhibition of the HPA axis. Dexamethasone will A. bind to the glucocorticoid receptors. B. promote binding of cortisol to the glucocorticoid receptor. C. mimic cortisol and result in increased release of ACTH. D. mimic cortisol and result in increased release of CRH. E. bind to the ACTH receptor.

C Cell surface receptors have ligand binding pocket on the extracellular side of the receptor. Thus, the transmembrane domains relay the signal to the intracellular domain of the cell surface receptors. In A, an antagonist is essentially an inhibitor. In B, the G-proteins that are coupled to the GPCRs have GTPase activity. In D, nuclear receptors can be present either in the cytosol or in the cell nucleus. In E, G-proteins, but not G-protein coupled receptors, can bind GTP molecules in their active sites.

Find ONE CORRECT statement. A. A hormone antagonist works like a natural hormone in signaling. B. Nuclear receptors have GTP hydrolysis activity. C. Cell surface receptors transmit the signal using transmembrane domains. D. Nuclear receptors are present only in the cell nucleus. E. All G-protein coupled receptors can bind GTP in their active sites.

D

Gleevec (Imatinib) is used to treat the hyperactive kinase of Bcr-Abl in patients with: A. Kaposi's sarcoma. B. cervical carcinoma. C. Burkitt's lymphoma. D. chronic myelogenous leukemia. E. hepatocellular carcinoma.

B, D, F, G These steps involve enzymatic reactions. In A, Glucagon binding to the receptor is a 1:1 binding and it triggers the signal but does not amplify the signal. Likewise, in C, activation of the adenylate cyclase by activated G-protein is 1:1 protein association and it is only a switch. In E, four cAMP molecules are required to activate two catalytic subunits of protein kinase A. The signal is thus not amplified at this step.

Glucagon signaling is mediated by the G-protein coupled receptor. One hormone bound to the receptor can trigger the release of tens of thousands of glucose molecules (G1P) from the glycogen. Which step amplifies the hormone signal? Find All the answers. A. Binding of glucagon to the receptor. B. Activation of heterotrimeric G-proteins by the receptor. C. Activation of adenylate cyclase by the G-protein. D. Production of cAMP by adenylate cyclase. E. Activation of protein kinase A by cAMP. F. Activation of glycogen phosphorylase kinase. (GPK) by protein kinase A. G. Activation of glycogen phosphorylase by GPK.

A

Haploinsufficiency refers to: A. loss-of-function mutations that cause some autosomal dominant diseases. B. loss-of-function mutations that cause some autosomal recessive diseases. C. gain-of-function mutations that cause some autosomal dominant diseases. D. gain-of-function mutations that cause some autosomal recessive diseases. E. meiotic instability during gametogenesis.

C Insulin receptor does not phosphorylate the Ras protein so A is wrong. PLCβ is not associated with the insulin signaling (B is wrong). The tyrosine residues are phosphorylated on the insulin receptor which recruits the IRS-1 protein via its phosphotyrosine recognition domain. Mutation in the domain of IRS-1 can result in failure to its interaction with the insulin receptor, thus C is correct.

In a comparative study of two related cell lines, you find that one responds normally to insulin while the other has an impaired response. You discover that both cell lines bind insulin with equal affinity, but that the impaired response is manifest in an inability to recruit the insulin receptor substrate-1 (IRS-1) protein to the receptor. This would most likely be due to which of the following? A. inability of the receptor to phosphorylate the Ras protein B. loss of activation of phospholipase C-β (PLCβ) C. mutation in the tyrosine phosphate recognition site of IRS-1 D. serine phosphorylation of the insulin receptor preventing IRS-1 binding E. tyrosine phosphorylation of the insulin receptor, leading to the loss of the IRS-1- binding site

C Granzyme B is a serine protease found in CTLs that can directly trigger apoptosis. CTLs express Fas ligand on their surfaces, and when contacting Fas receptors on the target cell, the ligand can induce apoptosis by the extrinsic (death receptor-initiated) pathway. BCL2 favors cell survival. Nitric oxide helps destroy phagocytized microbes. Endonu- cleases are generated following caspase activation and lead to nuclear fragmentation. When p53 is activated by intrinsic DNA damage during cell proliferation, apoptosis is triggered. Mutations in p53 may allow accumulation of genetic damage, a process that promotes unregulated cell growth (neoplasia).

In a study of viral hepatitis infection, it is observed that cytotoxic T lymphocytes (CTLs) induce death in virally infected hepatocytes. The CTLs release perforin to allow entry of their granules. Which of the following substances is found in those granules that directly activates programmed cell death? A BCL2 B Endonuclease C Granzyme B D Nitric oxide E p53

E Chromosomal telomere shortening in normal human cells limits their replicative potential and gives rise to rep- licative senescence. This occurs because most somatic cells lack the enzyme telomerase. Normal human stem cells do express telomerase. By contrast, 90% or more of human tu- mor cells show activation of telomerase, explaining contin- ued tumor growth in the body and "immortalized" cell lines in culture. All other pathways listed cannot affect telomerase shortening, which is the rate-limiting step in indefinite repli- cation of cells.

In an experiment, cells from human malignant neoplasms explanted into tissue culture medium continue to replicate. This allows development of "immortal" tumor cell lines that are extremely useful for the study of tumor biology and responses to therapeutic modalities. Activation of which of the following molecular components is most likely to endow these tumor cells with limitless replicative ability in vivo and in vitro? A Hypoxia-induced factor 1 B BCL2 gene C Cyclin-dependent kinase gene methylation D DNA replication repair E Telomerase

E. RAS proteins transduce signals from growth factor receptors, such as epidermal growth factor, that have intrinsic tyrosine kinase activity. G proteins perform a similar function for G protein-linked, seven-transmembrane receptors. Cyclic AMP is an effector in the G protein signaling pathway. Cyclins and and cyclin-dependent kinases regulate the cell cycle in the nucleus.

In an experiment, release of epidermal growth factor into an area of denuded skin causes mitogenic stimulation of the skin epithelial cells. Which of the following proteins is most likely to be involved in transducing the mitogenic signal from the epidermal cell membrane to the nucleus? A. Cyclic AMP B. Cyclin D C. Cyclin-dependent kinase D. G proteins E. RAS proteins

C

In regards to development in the fetus, which of the following statements is TRUE A. Meconium does not accumulate in the lower Ileum and colon B. The fetus does not swallow amniotic fluid C. The fetus has an alternative heart valve and body blood vessel system from that of the adult D. The uptake of amniotic fluid does not contribute to the protein needs of the fetus during its development.

D

In the above pedigree, what is the probability that individual III-4 is a carrier. A. nearly 100% B. 50% C. 33% D. 25% E. Nearly 0%

D

In the figure below which of the following is true about the relationship between A and B? A. B is an antagonist whereas A is an agonist B. B and A are the same drug, but B is administered at a lower rate than A C. B is less toxic than A D. B is a partial agonist with high receptor affinity; A is a full agonist with low receptor affinity E. B is a full agonist with low receptor affinity; A is a full agonist with high receptor affinity

A

In the figure below, Drug A represents an agonist that reduces blood pressure in hypertensive patients. Which of the following are true about Drug B? A. Drug B decreases the potency of Drug A B. Drug B decreases the efficacy of Drug A C. Drug B increases the potency of Drug A D. Drug B increases efficacy of Drug A E. Drug B does not modify the effects of Drug A

D

In the figure below, the graded dose-response curves represent the effects and side-effects of vasodilatix, a novel vasodilator, designed as anti-hypertensive medication in premenopausal women. The maximum therapeutic effect is a 40 mm reduction in blood pressure. For it to be more efficacious than simple change in life-style, it needs to be administered at the EC90. Which life-threatening side-effect(s) could occur at this high dose? (NOTE: read the graph carefully, including all the words in the legend). A. hypertrichosis and ankle swelling B. hypertrichosis and tachycardia C. hypertrichosis, tachycardia, and ankle swelling D. tachycardia E. tachycardia and ankle swelling

B. Primary oocytes are formed by month 5 of fetal life and remain dormant until puberty, when hormonal changes in the young woman stimulate the ovarian and menstrual cycles. From 5 to 15 oocytes will then begin maturation with each ovarian cycle throughout the woman's reproductive life.

In the production of female gametes, which of the following cells can remain dormant for 12 to 40 years? (A) Primordial germ cell (B) Primary oocyte (C) Secondary oocyte (D) First polar body (E) Second polar body

A. Primordial germ cells migrate from the wall of the yolk sac during week 4 of embryonic life and enter the gonad of a genetic male, where they remain dormant until puberty (about age 12 years), when hormonal changes in the young man stimulate the production of sperm.

In the production of male gametes, which of the following cells remains dormant for 12 years? (A) Primordial germ cell (B) Primary spermatocyte (C) Secondary spermatocyte (D) Spermatid (E) Sperm

E Autosomal recessive - loss of function, boy and girl affected, fully penetrant in the case of dominant mutations, parents unaffected. 1 x ½ x 1 x ½ = 1/4

Jack and Jill have a boy and a girl with cystinuria, though the parents are not affected. Cystinuria is caused by the loss-of-function of a transporter in the renal tubules whose function is to re-absorb a subset of amino acids including cysteine/cystine. Assuming complete penetrance in the case of dominant mutations, what is the likelihood that their next child will be affected by cystinuria? Assume that the carrier frequency for cystinuria is 1/10,000. A. 1/400,000,000 B. 1/4,000,000 C. 1/400,000 D. 1/400 E. 1/4

B

Jack, a 50-year-old male enters your clinic. He has difficulty with his speech, chewing and swallowing. He complains that he frequently chokes on food or liquids. John also complains of weakness in his arm and leg muscles. You observe some facial muscle weakness. What other condition is Jack likely to have? A. Mental retardation B. Gynecomastia (enlarged breasts) C. Frontal balding D. Bilateral ptosis (droopy eye lids) E. Grip myotonia

D 1 x ½ x 2/3 x ½ = 1/6

John has done genetic testing and learned that he is a carrier for phenylketonuria. Melissa has a brother with PKU, though she is not affected and has not been tested. Assuming that the population frequency of PKU is 1/60, what is the probability that their first child of John and Melissa will be affected by PKU? A. 1/240 B. 1/120 C. 1/60 D. 1/6 E. ¼

A

Knockout mice demonstrate the effect on survival of deleting one (p53+/-) or both (p53+/+) alleles of p53 (right). In humans, which disease illustrates the effect of deleting one p53 allele (p53+/-)? A. Li-Fraumeni disease B. Burkitt's lymphoma C. Chronic myelogenous leukemia D. Retinoblastoma E. Cervical cancer

D

Ms. DeeJee Oxin is a 65-year-old woman who has been diagnosed with congestive heart failure from a recent myocardial infarction. She is prescribed a high dose (1 mg) of digoxin (a cardiotonic drug), and she is asked to visit the doctor's office for a follow-up one week later. During her follow-up, she shows clear signs of digitalis toxicity. Plasma digoxin levels are 4 ng/mL, whereas therapeutic levels are about 1 ng/mL. Creatinine levels are 1 mg/dL (normal range: 0.6-1.4 mg/dL), and the half life for this drug in this patient is 1.5 days. If, instead of withholding treatment, you had simply switched the dose of digoxin to the new one, how many days would it have taken to reach steady state concentration of the drug? A. 3 days B. 4 days C. 5 days D. 7 days E. 10 days

C

Ms. DeeJee Oxin is a 65-year-old woman who has been diagnosed with congestive heart failure from a recent myocardial infarction. She is prescribed a high dose (1 mg) of digoxin (a cardiotonic drug), and she is asked to visit the doctor's office for a follow-up one week later. During her follow-up, she shows clear signs of digitalis toxicity. Plasma digoxin levels are 4 ng/mL, whereas therapeutic levels are about 1 ng/mL. Creatinine levels are 1 mg/dL (normal range: 0.6-1.4 mg/dL), and the half life for this drug in this patient is 1.5 days. You decide to stop treatment so as to reduce drug concentration levels. Approximately how long should you withhold treating your patient for therapeutic levels to be reached? A. 1 day B. 1.5 days C. 3 days D. 4.5 days E. 6 days

D Single nucleotide polymorphisms (SNPs) are found in less than 0.5% of the genome, and only 1% of these are found in coding regions that affect protein synthesis. Some of these account for point mutations that may be associated with disease conditions. C number variations (CNVs) in- volve variations in large contiguous regions of DNA from 1000 to a million base pairs. Epigenetic changes involve modulation of gene expression without any change in the DNA. Trinucleotide repeats involve increased numbers of base pairs. RNA alterations may modulate DNA expres- sion, such as noncoding micro RNAs.

Multiple members of a family have a disease that is associated with a genetic change that involves a substitution of adenine for thymine involving one base pair on homologous chromosomes. What is the best term to describe this finding? A Copy number variation B Deletion C Epigenetic change D Single nucleotide polymorphism E Trinucleotide repeat mutation F RNA alteration

B My explanation (unsure if this is correct): If the choline is not transported by Na/H exchanger, then H+ would remain intracellularly. Therefore, you will have an acidification. Then, looking at the original out & in concentrations of Na, they are different by 10 fold.

Nao = 140 mM Nai = 14 mM Ko = 16 mM Ki = 160 mM Vm = -60 mM Clo =200 mM Cli = 20 mM pHo = 7.4 pHi = 7.3 Assuming that there is a 1Na+ /1H+ exchanger (counter-transporter, antiporter) responsible for maintaining the intracellular pH, what effect on pHi would the isosmotic replacement of extracellular Na+ by a cation (e.g., choline) which is not transported by the Na/H exchanger have on the intracellular pH? A. 10 fold alkalinization B. 10 fold acidification C. 100 fold alkalinization D. 100 fold acidification

A My explanation (unsure if this is correct): m=# of Na+ ions n= #of net (+) charges In this case m=4, n=2 10^4 * 10^2 = 10^6

Nao = 140 mM Nai = 14 mM Ko = 16 mM Ki = 160 mM Vm = -60 mM Clo =200 mM Cli = 20 mM pHo = 7.4 pHi = 7.3 What would be the Mg2+ gradient that a 4Na+ /1Mg2+ exchanger (antiporter) could establish in the above cell? A. 10^6 fold B. 10^3 fold C. 10^2 fold D. 10 fold

A My explanation (unsure if this is correct): Number of ions: 1 Net charge: 0 10^1 * 10^0 = 10

Nao = 140 mM Nai = 14 mM Ko = 16 mM Ki = 160 mM Vm = -60 mM Clo =200 mM Cli = 20 mM pHo = 7.4 pHi = 7.3 What Cl gradient would a 1K+ /1Cl cotransporter (symporter) be able to establish in the above cell? A. 10 fold B. 100 fold C. 1000 fold D. 10,000 fold

C My explanation (unsure if this is correct): m=# of Na+ ions n= #of net (+) charges In this case m=2, n=1 10^2 * 10^1 = 10^3 = 1000 fold

Nao = 140 mM Nai = 14 mM Ko = 16 mM Ki = 160 mM Vm = -60 mM Clo =200 mM Cli = 20 mM pHo = 7.4 pHi = 7.3 What would be the concentrating ability of a 2Na+ /1aa cotransporter (symporter) if it operates with a stoichiometry of 2Na+ per 1 aa-1 and the aa has a valence of -1? A. 10 fold B. 100 fold C. 1000 fold D. 10,000 fold

E

No Alki, a 23-year-old Japanese male, is invited for the opening ceremony of an art show. During the show, he and his European friend Hedo Nist eat small appetizers, smoke several cigarettes, and drink a couple of glasses of Champagne. While Hedo Nist seems to handle it well, No Alki shows intense facial flushing, slight sweating, and mild signs of ethanol intoxication. This phenomenon MOST LIKELY occurred to No Alki because: A. cigarette smoking has decreased ethanol metabolism B. cigarette smoking has decreased ethanol renal clearance C. ethanol conjugation is impaired in this person D. ethanol is metabolized too slowly, with first order kinetics E. genetic variations of enzymes in the ethanol metabolic pathway

A The onset of menstruation is orderly, programmed cell death (apoptosis) through hormonal stimuli, an ex- ample of the intrinsic (mitochondrial) apoptotic pathway. As hormone levels drop, the endometrium breaks down, sloughs off, and then regenerates. With cellular atrophy, there is often no visible necrosis, but the tissues shrink, something that occurs in the endometrium after menopause. Caseous necrosis is typical of granulomatous inflammation, resulting most commonly from mycobacterial infection. Heterophagocytosis is typified by the clearing of an area of necrosis through macrophage ingestion of the necrotic cells. Liquefactive necrosis can occur in any tissue after acute bacterial infection or in the brain after ischemia.

On day 28 of her menstrual cycle, a 23-year-old woman experiences onset of menstrual bleeding that lasts for 6 days. She has had regular cycles since menarche. Which of the following processes most likely occurs in her endometrial cells to initiate the onset of menstrual bleeding? A Apoptosis B Atrophy C Caseous necrosis D Heterophagocytosis E Liquefactive necrosis

E

Once a drug binds to metabotropic receptors, which of the following statements is MOST CORRECT? A. receptors become enzymatically active B. receptors undergo conformational changes to allow ion flow C. receptors translocate in the nucleus, where they serve as transcription factors D. receptors bind tyrosine kinase and phosphorylate downstream signaling proteins E. receptors trigger dissociation of G proteins into an alpha and a beta-gamma dimer

B

Ozzy, a 70kg narcotic addict is brought to the emergency room with signs of severe respiratory depression. He claims to have injected a large dose of morphine 3 hours earlier. Immediate blood analysis shows morphine blood levels of 0.5 mg/L. Assuming that the pharmacokinetics of morphine are Vd 200L, and t ½ = 3 hours, how much morphine did Ozzy inject? A. 100 mg B. 200 mg C. 300 mg D. 400 mg E. 500 mg

D Css=3 µg/mL CL=100 mL/h maintenance dose=Css*CL =3 µg/mL * 100 mL/h =300 µg/hr =0.3 g/hr =3 g/10 hr

Ruprecht is an elderly man who weighs 70 kg. He is admitted to the hospital with a severe upper and lower urinary tract infection. A broad-spectrum antibiotic is to be administered as a dose of 3 mg (i.e. 3000 µg). Given that the clearance rate for this antibiotic is 100 mL/h, every how often should you administer this dose (3 mg) if you want to remain within a therapeutic window of about 3 µg/mL? A. 1 hour B. 3 hours C. 6 hours D. 10 hours E. 30 hours

7

Sandra has a long family history of glucose-6-dehydrogenase deficiency. Patients with this disease predisposes one to anemia due to the hemolysis of the red blood cells. 400 million people globally have this condition. You take a family history and obtain the pedigree below. How many obligate carriers are there in this pedigree?

D DAG & IP3 are the 2nd messengers. PKC is activated by diacylglycerol and calcium. PIP2 is the substrate of the PKC, thus it is not a second messenger. PIP3 is involved in activation of Akt (protein kinase B).

T-cells are activated when antigen-MHC complexes bind to the T-cell receptor on the cell membrane. This process activates second messengers that in turn activate protein kinase C (PKC), leading to the induction of transcription factors and subsequent gene expression. Which of the following most likely represents the second messenger of this signal transduction? A. PIP2. B. cAMP C. cGMP D. diacylglycerol E. PIP3

B

Telomerase is inactive in virtually all somatic cells. In contrast, telomerase is active in 85% to 90% of human tumor cell samples. This activity provides a selective advantage for proliferating cells by providing a mechanism for: A. preventing translation. B. preventing telomere shortening. C. adding telomeres to the 5' ends of DNA. D. promoting cell death. E. promoting transcription

C

The Philadelphia chromosome is associated with chronic myelogenous leukemia. There is a translocation between chromosome 9 and 22, t(9;22)(q34;q11) that creates a chimeric gene between BCR-Abl on chromosome 22. Abl is a tyrosine kinase that because of the translocation is under the control of the BCR gene and thus constitutively active. This mutation is best described as a: A. loss of function mutation B. dominant negative mutation C. gain-of-function (activity) mutation D. variable expressing mutation E. heterogeneous mutation

D

The U.S. African American population has an incidence of 1 in 400 for sickle cell anemia. See a typical pedigree (see Figure). Assuming Hardy-Weinberg equilibrium (p2 + 2pq + q2 = 1; p + q = 1), what is carrier frequency of this disease in this population? A. 0.001 B. 0.01 C. 0.05 D. 0.095 E. 0.95

C

The classic cancer drugs often have many toxicities associated with them for which of the following reasons? A. Because they were designed by older methods of drug design. B. Because of the genetic variants we all carry. C. Because they target all actively proliferating cells. D. Because they target cell of the digestive system.

A

The figure below represents dose-response curves for a new drug used to treat tachycardia. The therapeutic effects consist in a decrease in heart rate (maximal effect is a decrease in heart rate by 100 beats per minute). Its side-effects consist in hypotension (maximal effect is a decrease in blood pressure by 100 mg/Hg). Both therapeutic effects (solid lines) and side-effects (hashed lines) are evaluated in three different patients (Patient a, b, or c). These patients have heart beats of 80, 90, and 100 beats per minute respectively. If you would like to reach 70 beats per minute in all patients, which of the following best describes the manner in which blood pressure will be lowered across patients? A. <2 mm/Hg in patients (a), (b), (c) B. ~5 mm/Hg in patient (a) C. ~30 mm/Hg in patient (a) D. ~5 mm/Hg in patient (a) and ~2 mm/Hg in patient (b) E. ~30 mm/Hg in patient (a) and ~10 mm/Hg in patient (b)

D

The figure below represents the plasma concentrations of pharmagood, a new drug developed to promote critical thinking. Pharmagood is given as a bolus intravenous injection of 2048mg in a normal individual (a pharmacology student, weighing 70kg). The empty circles represent plasma concentrations of pharmagood in its alpha phase, whereas the filled circles represent concentrations of pharmagood during its beta phase (before or after the administration of sodium bicarbonate). What is the approximate clearance rate before the administration of sodium bicarbonate? A. 0.175 L/h B. 0.35 L/h C. 0.7 L/h D. 1.4 L/h E. 2.8 L/h

E

The figure below shows the effects of three different drugs (X, Y, or Z) on adenylyl cyclase production. Each drug is administered on its own. Which of the following is true? A. X is a full agonist, Y is a partial agonist, and Z is an antagonist B. X is a full agonist, Y is an antagonist, and Z is a partial agonist C. X is a full agonist, and Y and Z are partial agonists D. X is a full agonist, and Y and Z are antagonists E. X and Y are full agonists, and Z is a partial agonist F. X and Y are full agonists, and Z is an antagonist

C This represents a net loss of water from both compartments (ECF, ICF), upon equilibration.

The following Darrow-Yannet diagrams represent alterations of body fluid compartments, indicated by the dashed lines describing the altered conditions relative to the continuous lines showing normal conditions. Volume is shown on the horizontal axis, and osmolarity is depicted on the vertical axis. The left compartment is ECF; the right compartment is ICF. Dehydration characterized by net loss of water from total body fluids is illustrated by which of the Darrow-Yannet diagrams shown above (A - E)?

C

The graphs below show the effect of grapefruit juice or orange juice on the graded dose-response curve for an antihistaminic prescription drug (Drug A), which is frequently used to treat seasonal allergies. The curves represent the average population response for the drug. Drug A has the favorable feature that it produces relatively little sedation (a frequent and unwanted side effect of this class of drugs), but at higher doses it has been found to produce serious cardiac toxicities. Given the information above, which statement is correct? A. grapefruit juice decreases the potency of the antihistamine drug B. grapefruit juice likely increases absorption of drug A C. orange juice is a safer alternative to grapefruit juice for people taking the antihistamine drug D. people taking the antihistamine drug should drink a lot of grapefruit juice, as it increases the efficacy of the antihistamine drug E. people taking the antihistamine drug should drink a lot of grapefruit juice, as it increases the potency of the antihistamine drug

D

The homeostasis in the living organism requires energy for: A) maintenance of equilibrium. B) water diffusion through cellular membranes. C) oxygen diffusion through alveolar membranes. D) maintenance of steady state.

D - often seen in hypertension heart disease

The illustration shows a section of the heart from a 45-year-old African-American man with longstanding hypertension who died of a "stroke." Which of the following adaptive changes is exemplified in the illustration? (A) Aplasia (B) Atrophy (C) Hyperplasia (D) Hypertrophy (E) Hypoplasia

A

The initial (starting) state of a closed system (cells and bath) is indicated below. σ refers to the reflection coefficient for each solute. Assume that the cells can survive any shrinkage or swelling that may occur. A. The cells will shrink initially, and their final volume will be 400ml. B. The cells will swell initially, and their final volume will be 400ml. C. The cells will shrink initially, and their final volume will be 653ml. D. The cells will swell initially, and their final volume will be 653ml. E. The cells will shrink initially, and their final NaCl concentration will be 150mM.

B

The log kill hypothesis refers to which of the following statements with regards to cancer treatment? A. Drugs act via first-order kinetics killing all cells. B. Drugs act via first-order kinetics killing a constant proportion of a cell population. C. Drugs act via second-order kinetics requiring 2 drugs to kill all cells. D. Drugs act via second-order kinetics requiring 2 drugs to kill a constant proportion of a cell population.

B

The male incidence of color blindness is 8%. Assume Hardy-Weinberg equilibrium (p2 + 2pq + q2 = 1; p + q = 1). What is the frequency of the color blindness allele in the male population? A. 0.04 B. 0.08 C. 0.20 D. 0.25 E. 0.50

B Sinusoidal capillaries found in bone marrow, liver, spleen. You can see bile ducts & sinusoidal capillaries in this image, indicative of hepatocytes (liver) which partially function to detoxify toxic compounds & drugs.

The organ shown at low and high magnification functions to A. aid in respiration. B. detoxify toxic compounds and drugs. C. transport food in the digestive system. D. break down food through producing digestive enzymes

E

The tracings below represent recordings from a collection of excitable Na channels during a step change in voltage to a membrane potential above threshold. The red circled deflection represents: A. a single sodium ion traversing the Na channel from outside to inside the axon. B. a single sodium ion traversing the Na channel from inside to outside the axon. C. non-inactivating inward sodium current. D. inactivating outward sodium current. E. many sodium ions/sec entering the axon.

F

This disease is inherited as an X-linked dominant manner A. Tay Sachs B. Huntington's disease C. Familial hypercholesterolemia D. Neurofibromatosis E. β-thalassemia F. Rett syndrome G. Muscular dystrophy H. Cystic Fibrosis I. 45, X J. 47, XXY K. 47, XXX L. 47, XYY M. 47, XY, +13 N. 47, XX, +18 O. 47, XY, +21

D

This disease demonstrates complete penetrance, variable expressivity, and is inherited in an autosomal disease manner A. Tay Sachs B. Huntington's disease C. Familial hypercholesterolemia D. Neurofibromatosis E. β-thalassemia F. Rett syndrome G. Muscular dystrophy H. Cystic Fibrosis I. 45, X J. 47, XXY K. 47, XXX L. 47, XYY M. 47, XY, +13 N. 47, XX, +18 O. 47, XY, +21

A

This disease is a lysosomal Storage disease A. Tay Sachs B. Huntington's disease C. Familial hypercholesterolemia D. Neurofibromatosis E. β-thalassemia F. Rett syndrome G. Muscular dystrophy H. Cystic Fibrosis I. 45, X J. 47, XXY K. 47, XXX L. 47, XYY M. 47, XY, +13 N. 47, XX, +18 O. 47, XY, +21

G

This disease is an X-linked recessive disease A. Tay Sachs B. Huntington's disease C. Familial hypercholesterolemia D. Neurofibromatosis E. β-thalassemia F. Rett syndrome G. Muscular dystrophy H. Cystic Fibrosis I. 45, X J. 47, XXY K. 47, XXX L. 47, XYY M. 47, XY, +13 N. 47, XX, +18 O. 47, XY, +21

B

This disease is characterized by an increase incidence and severity with successive generations. A. Tay Sachs B. Huntington's disease C. Familial hypercholesterolemia D. Neurofibromatosis E. β-thalassemia F. Rett syndrome G. Muscular dystrophy H. Cystic Fibrosis I. 45, X J. 47, XXY K. 47, XXX L. 47, XYY M. 47, XY, +13 N. 47, XX, +18 O. 47, XY, +21

J. Klinefelter's Syndrome

This disease is characterized by hypogonadism, taller and longer arms, and most are sterile. A. Tay Sachs B. Huntington's disease C. Familial hypercholesterolemia D. Neurofibromatosis E. β-thalassemia F. Rett syndrome G. Muscular dystrophy H. Cystic Fibrosis I. 45, X J. 47, XXY K. 47, XXX L. 47, XYY M. 47, XY, +13 N. 47, XX, +18 O. 47, XY, +21

B Loss of nuclei, but preservation of myocardial architecture.

This figure illustrates the microscopic appearance of the heart of a 56-year-old man who died after a 24- hour hospitalization for severe "crushing" chest pain complicated by hypotension and pulmonary edema. The type of necrosis shown is best described as

A

This syndrome is characterized by severe hypotonia and feeding difficulties in early infancy, followed in later infancy or early childhood by excessive eating and gradual development of morbid obesity (unless eating is externally controlled). Motor milestones and language development are delayed. All individuals have some degree of cognitive impairment. This disease is caused by A. del 15[q11q13] pat or UPD (mat) B. del 15[q11q13] mat or UPD (pat) C. del 15[q11q13] mat or UPD (mat) D. del 15[q11q13] pat or UPD (pat) E. exclusively del 15[q11q13] mat

The answer is E: Nuclear fragmentation. Nuclear fragmentation (karyorrhexis and karyolysis) is a hallmark of coagulative necrosis. Choices A, B, and D are incorrect because they are features of both reversibly and irreversibly injured cells. Lymphocytes (choice C) are a hallmark of chronic inflammation.

What would you see in coagulative necrosis? (A) Disaggregation of polyribosomes (B) Increased intracellular volume (C) Influx of lymphocytes (D) Mitochondrial swelling and calcifi cation (E) Nuclear fragmentation

E. At birth, a male has primordial germ cells in the testes that remain dormant until puberty, at which time they differentiate into type A spermatogonia. At puberty, some type A spermatogonia differentiate into type B spermatogonia and give rise to primary spermatocytes by undergoing DNA replication.

When does formation of primary spermatocytes begin? (A) During week 4 of embryonic life (B) During month 5 of fetal life (C) At birth (D) During month 5 of infancy (E) At puberty

A

Which WEAKLY ACIDIC drug is distributed more readily from the blood (pH 7.4) to the cerebrospinal fluid (pH 7.3)? A. Drug A; pKa=7.6 B. Drug B; pKa=7.5 C. Drug C; pKa=7.4 D. Drug D; pKa=7.3 E. Drug E; pKa=7.2

E

Which WEAKLY BASIC drug is absorbed more readily from the LARGE INTESTINE? (l/w = lipid/water partition coefficient) A. Drug A; pKa=9, l/w=1 B. Drug B; pKa=9, l/w=100 C. Drug C; pKa=7, l/w=100 D. Drug D; pKa=1, l/w=1 E. Drug E; pKa=1, l/w=100

E

Which WEAKLY BASIC drug is absorbed more readily from the STOMACH? (l/w = lipid/water partition coefficient) A. Drug A; pKa=9, l/w=1 B. Drug B; pKa=9, l/w=100 C. Drug C; pKa=7, l/w=100 D. Drug D; pKa=1, l/w=1 E. Drug E; pKa=1, l/w=100

E

Which WEAKLY BASIC drug is distributed more readily from the blood (pH 7.4) to the cerebrospinal fluid (pH 7.3)? A. Drug A; pKa=7.6 B. Drug B; pKa=7.5 C. Drug C; pKa=7.4 D. Drug D; pKa=7.3 E. Drug E; pKa=7.2

A

Which of the following BEST describes drug metabolites? A. they are a consequence of phase I or phase II metabolism B. they are generally toxic C. they are less readily excreted D. they are transported by plasma proteins E. they represent the inactive form of the drug

D

Which of the following BEST describes the generic name of a drug? A. it costs millions of dollars to be created B. it is created based on the drug's natural, pure, semi-synthetic, or synthetic origin C. it refers to the chemical name of the drug D. it refers to the non-proprietary name of the drug E. it refers to the trade name of the drug

B. The syncytiotrophoblast plays the most active role in invading the endometrium of the mother's uterus. During the invasion, endometrial blood vessels and endometrial glands are eroded and a lacunar network is formed.

Which of the following components plays the most active role in invading the endometrium during blastocyst implantation? (A) Epiblast (B) Syncytiotrophoblast (C) Hypoblast (D) Extraembryonic somatic mesoderm (E) Extraembryonic visceral mesoderm

C Postganglionic neurons of the parasympathetic nervous system release ACh that binds to muscarinic receptors. In smooth muscle ACh causes contraction via M1-Gq-IP3-Ca2+ pathway. ACh slows the heart rate via M2-K+ channel pathway.

Which of the following correctly characterizes postganglionic neurons of the parasympathetic nervous system? A. Target tissues are activated through adrenergic receptors. B. Target tissues are activated through nicotinic receptors. C. Target tissues are activated through muscarinic receptors. D. Stimulation of these neurons causes bronchial smooth muscle to relax. E. Stimulation of these neurons causes the heart to beat faster.

D

Which of the following describes a concern that must be taken into consideration about Pharmacogenomics? A. Pharmacogenetic profiling may lead to increased cost of medical care B. Pharmacogenetic profiling may put an individual's privacy at risk C. Pharmacogenetic profiling may provide more information than originally needed, thus raising the question of whether the health care provider should provide this additional information to the patient D. All of the above

E. Cleavage is a series of mitotic divisions by which the large amount of zygote cytoplasm is successively partitioned among the newly formed blastomeres. Although the number of blastomeres increases during cleavage, the size of individual blastomeres decreases until they resemble adult cells in size.

Which of the following events is involved in the cleavage of the zygote during week 1 of development? (A) A series of meiotic divisions forming blastomeres (B) Production of highly differentiated blastomeres (C) An increased cytoplasmic content of blastomeres (D) An increase in size of blastomeres (E) A decrease in size of blastomeres

C In A, the nuclear receptors amplify hormone signals by mRNA and protein synthesis. In B, the G-proteins coupled to the GPCRs are GTPases. C is correct since the EGF receptor and insulin receptor require ATP for phosphorylation of the receptors for activation. D is wrong since pertussis toxin is an ADP-ribosylase, not a transcription factor. In E, β-Arrestins desensitize G-protein coupled receptors.

Which of the following is a CORRECT statement? A. The nuclear receptors amplify hormone signals by DNA replication. B. The G-protein coupled receptors (GPCRs) are GTPases. C. The EGF receptor and insulin receptor require ATP for receptor activation. D. Pertussis toxin is a transcription factor. E. β-Arrestins desensitize receptor tyrosine kinases.

B

Which of the following is the BEST example of a genetic disorder due to a mutation that acts in a dominant negative manner? A. Achondroplasia B. Huntington's disease C. Rett syndrome D. Neurofibromatosis E. Familial hypercholesterolemia (FH)

A

Which of the following list best illustrates the idea of increasing levels of complexity? A) Cells, tissues, organs, systems B) Tissues, cells, organs, systems C) Organs, systems, cells, tissues D) Systems, cell, tissues, organs

E

Which of the following phenotypes pictured below is caused by a mutation that is paternally inherited?

D Parasympathetic system activation is associated with rest and digest. Thus, the stimulation of the parasympathetic system results in increased GI motility, reduced blood pressure, bladder emptying, reduced heart rate and also increased salivary secretion (good for digestion). ACh causes bronchial smooth muscle contraction. This is why an anticholinergic drug is used to relieve symptoms of asthma. ACh reduces pupil size - hence atropine (a muscarinic receptor blocker) is used by opthalmologists to dilate the pupil to allow visual inspection of the interior of the eye. Large pupil size is a sympathetic response.

Which of the following physiological responses is most likely to be observed when the parasympathetic nervous system is stimulated in a healthy person? A. Pulmonary smooth muscle relaxation B. Reduced gastrointestinal motility C. Elevated blood pressure D. Elevated salivary secretion E. Pupil dilation (mydriasis)

E Remember the FFF (fight-flight-fright) response of the sympathetic stimulation. What physiological responses would you wish for when you are experiencing FFF? The body would require more oxygen for increased metabolic activity and bronchodilation should help to bring in more air. Increased heart rate and contractility should also help elevate cardiac output for supplying more oxygen. Vasoconstriction would occur (particularly in skin, digestive tract) and allow more blood to be present in the muscle. You would not want to urinate during FFF - less frequent urination. Sympathetic system dilates pupil size, a response to stressful stimuli.

Which of the following physiological responses is most likely to be observed when the sympathetic nervous system is stimulated in a healthy person? A. Bronchoconstriction B. Bradycardia C. Hypotension D. Frequent urination E. Pupil dilation (mydriasis)

D Correct signaling pathways are: α1 -Gq -PLC -IP3; IP3 elevates [Ca2+] and causes contraction. M2-Gi -Activation of KACh channel causes slowing of heart rate β1 -Gs -AC - cAMP - activates IF; increase Na+ influx and heart rate. β2 -Gs -AC - cAMP - PKA -MLCK; leads to smooth muscle relaxation. M1/3 -Gq - PLC -IP3; IP3 elevates [Ca2+] and causes contraction.

Which of the following receptors and physiological responses are correctly paired with regard to the autonomic nervous system function? A. α1 - vascular smooth muscle relaxation B. M2 - increase in heart rate C. β1 - decrease in heart rate and contractile force D. β2 - vascular smooth muscle relaxation E. M1/M3 - gastrointestinal smooth muscle relaxation

C

Which of the following statements BEST describes the passage of acidic drugs through membranes? A. it depends mostly on the pH of the drug B. it depends mostly on the thickness of the membrane the drug has to cross C. it is influenced by the relationship between the pH of the environment and the pKa of the drug D. it occurs more readily when the drug is at the same pH as its pKa E. it usually requires the presence of active transport or co-transport

D

Which of the following statements is FALSE regarding Spermatogenesis a. Primary germ cells in the male embryo lead to Spermatogonia in the developing testis b. Spermatogonia will develop eventually into spermatids c. Spermatids are immobile d. Development of the spermatid to the mature sperm involves Meiosis

D

Which of the following statements is FALSE regarding embryonic folding a. Cranial-caudal folding is important for repositioning of structures such as the heart, to it final position in the embryo b. Cranial-caudal folding involves bending of the cranial and caudal regions of the embryo toward the ventral slide of the embryo c. Lateral folding results in expansion of the amniotic cavity to the right and left side of the embryo (ie encircling the embryo with the amniotic sac) d. Lateral folding does not involve formation of the pleural or peritoneal membranes

B. Spina bifida involves failure of the caudal neurotube to close.

Which of the following statements is FALSE regarding neurulation a. The final phase of neurotube closure involves sealing of the cranial and caudal neuropores b. Spina bifida involves failure of the cranial neurotube to close c. Closure of the neurotube involves adhesion of neuro-ectoderm cells d. At the time the neural plate fuses to form the neurotube, the non-neuronal ectoderm associated with it also fuses to form the future scalp that covers the skull

D

Which of the following statements is FALSE regarding the fertilization and early formation of the zygote a. More than one sperm can pass through the zona pellucida b. Fusion of the sperm with the plasma membrane of the egg triggers the egg to proceed through Meiosis II c. Pronuclei contain half the amount of DNA than that found in nuclei of diploid cells d. Sperm are not involved in triggering the "fast block" to polyspermia when they bind to the plasma membrane of the egg

B Neural crest cells are specialized ectoderm.

Which of the following statements is TRUE regarding derivatives of the three germ layers a. Endoderm will form skin b. Ectoderm will form the early central nervous system c. Mesoderm will form neural crest cells d. The embryonic mesoderm germ layer and the extraembryonic mesoderm are identical cell layers that have identical functions during development

D

Which of the following statements is TRUE regarding development during the fetal period a. Anterior Pituitary Gland becomes functional at birth b. The hypothalamus becomes functional at birth c. The brain sulci development is completed by the time of birth d. The skull bones do not completely fuse until after birth.

A. Primary oocytes are enclosed in follicles.

Which of the following statements is TRUE regarding development of oocytes in the ovaries a. Primary oocytes are surrounded by a plasma membrane and follicular cells b. Follicular cells cannot affect development (e.g. meiosis) of the developing oocyte c. Normally only one primary oocyte per month is stimulated to proceed through Meiosis I d. Following ovulation, the egg always enters the fallopian tube

A. The primitive pit is nearer to buccopharyngeal membrane.

Which of the following statements is TRUE regarding gastrulation a. The primitive streak forms in the epiblast layer b. The primitive node is positioned toward the pole of the embryo where the cloacal membrane is located c. The embryonic mesoderm forms from cells of the extraembryonic mesoderm d. The embryonic mesoderm germ layer arises from cells that originate from the hypoblast layer

A

Which of the following statements is teleological in nature: A) The purpose of the heart is to maintain the blood pressure. B) Activation of the sympathetic nervous system accelerates the heart rate. C) The pituitary gland is the "Master Gland" of the endocrine system. D) Bright light causes miosis (pupil constriction). Rational: Teleology assigns purpose to the organs of the body.

C. The entire cardiovascular system is of mesodermal origin.

Which of the three primary germ layers forms the histologically definitive endocardium of the adult heart? (A) Ectoderm (B) Endoderm (C) Mesoderm (D) Epiblast (E) Hypoblast

C A is wrong since the receptor tyrosine kinases phosphorylate tyrosine residues on the receptors. In B, steroid hormone signaling is slow. In D, neurohormonal cascade is mediated by the cell surface receptors and the nuclear receptors in many cases (e.g., HPA-axis, CRH binds to a GPCR but the cortisol acts on a nuclear receptor. In E, cell surface receptors have intracellular domains.

Which one in the following is a CORRECT statement? A. Receptor tyrosine kinases phosphorylate serine/threonine residues on the target proteins. B. Steroid hormone signaling is specific, fast and irreversible. C. Hormone signal transduction involves signaling amplification. D. Neurohormonal cascade is mediated only by the cell surface receptors. E. Cell surface receptors do not have intracellular domains.

D

Which one of the following conditions may NOT cause activation of a proto-oncogene leading to transformation and/or oncogenesis? A. a point mutation produces enhanced activity of a tyrosine kinase B. truncation of the extracellular domain of a receptor causes lack of regulation by ligand binding C. a promoter/enhancer mutation causes overproduction of a 'normal' transcription factor D. hyperactivation of a caspase leads to excessive cleavage of DNA E. reciprocal translocation within a chromosome produces gene fusion and a hyperactive oncoprotein

C Sympathetic cholinergic system is generally inactive at rest. Because the postganglionic neurons release ACh, it is named cholinergic even though the preganglionic neuron arises from the thoracolumbar portion of the spinal cord. Sweat glands are innervated by the sympathetic cholinergic neurons

Which one of the following correctly characterizes the sympathetic cholinergic system? A. It is normally active at rest at comfortable room temperatures. B. Norepinephrine is released from the postganglionic neuron. C. It innervates sweat glands of the skin. D. Its preganglionic neurons arise from the cranial portion of the nervous system. E. Norepinephrine is the neurotransmitter in its ganglionic synapses

C Sympathetic cholinergic system is normally inactive but becomes active under stressful situations. The transmitter released by the preganglionic and postganglionic neurons is ACh (not NE) and acts on the muscarinic receptors in the sweat glands. Sympathetic preganglionic neurons arise from the thoracic/lumbar regions of the spinal cord.

Which one of the following correctly characterizes the sympathetic cholinergic system? A. It is normally very active in resting people at comfortable room temperatures. B. Norepinephrine is released from the postganglionic neuron. C. It innervates sweat glands in the skin. D. Its preganglionic neurons arise from the cranial portion of the nervous system. E. Norepinephrine is the neurotransmitter in its ganglionic synapses

B Parasympathetic system is associated with rest and digest

Which one of the following effects is observed with increased parasympathetic nerve activity? A. Increased heart rate due to decreased pacemaker potential B. Increased motility of the gastrointestinal tract C. Increased pupil size D. Increased sweating E. Increased metabolism of skeletal muscle

D Nuclear receptos are transcription factors that can either activate or inhibit transcription (transactivation vs transrepression).

Which one of the following has a transcription activation domain? A. Adrenocorticotropic hormone (ACTH). B. adenylate cyclase. C. protein kinase A. D. glucocorticoid receptor. E. G-protein coupled receptor.

C All other choices are observed following stimulation of the parasympathetic system that is associated with conservation of energy, restoration of energy (rest and digets), and with local and discrete effects due to low pre-postganglionic neuron ratio (~1:3) and due to rapid degradation of ACh by the action of acetylcholinesterases in the plasma.

Which one of the following is most likely to be exhibited in response to increased activity of the sympathetic portion of the autonomic nervous system? A. Localized and discrete responses B. Conservation of energy C. Fight, fright and flight responses D. Decreased heart rate E. Increased gastrointestinal motility

C All adrenergic and muscarinic receptors are coupled to G proteins that then act on effectors.

Which one of the following statements regarding the adrenergic signaling pathway is most accurate? A. The adrenergic receptors bind acetylcholine. B. The adrenergic receptors act directly on effector molecules such as ion channels. C. GTP-binding proteins transduce the signal from the receptor to the effector. D. The adrenergic receptors are ion channels. E. The adrenergic receptors are part of the parasympathetic nervous signaling pathway.


Related study sets

Cognitive Psychology- Ch. 3 Practice Questions

View Set

Indiana Life and Health-Life Insurance Policies

View Set

WFG- Completing the Application, Underwriting, and Delivering the Policy

View Set

Path 370 Assessment 2 (CH. 10, 11, 13, 14, 15)

View Set

Pre-Lecture Quizzes: Peri-operative

View Set

mitosis/meiosis/mendelian genetics practice

View Set